Hand Nerves Flashcards

1
Q

Consultation is requested for a 7-year-old girl because of intravenous infiltration of a chemotherapeutic agent in the dorsal forearm. Physical examination shows firmness and swelling of the forearm and pain on passive flexion of the wrist. Which of the following is the most appropriate initial management?
A) Administration of an antidote
B) Doppler sonography of the forearm
C) Liposuction and saline flush of the affected area
D) Measurement of compartment pressures
E) Surgical excision and grafting

A

D) Measurement of compartment pressures

Extravasation usually remains localized, yet some patients develop necrotic problem wounds. Often initially underestimated, the extent of injury can declare itself widely with time. Compartment syndrome in an extremity extravasation should be initially ruled out either by clinical assessment or direct measurement of compartment pressures. Tissue loss can include skin, muscle, tendon, nerve, vasculature, and/or joint. Given the variable amount of soft-tissue involvement, early conservative therapy is recommended. Immediate discontinuation of the infusion at the affected site is paramount and should not be overlooked. Aspiration, liposuction, wound excision, debridement, grafts, flaps, and antidote administration have all been described in the management of extravasation injury.

How well did you know this?
1
Not at all
2
3
4
5
Perfectly
2
Q
A 24-year-old man comes to the office because of numbness and difficulty moving his ring and little fingers 5 months after cutting his upper arm on broken glass. Current physical examination shows inability to abduct and adduct the ring and little fingers. Sensation to light touch is diminished. Following exploration and resection of a painful, traumatic neuroma, there is a 5-cm gap in the ulnar nerve proximal to the elbow. Which of the following is the most appropriate management to restore intrinsic muscle function?
A) Cadaveric nerve allografting
B) Nerve transfer
C) Sural nerve grafting
D) Use of nerve conduit
E) Vascularized nerve grafting
A

B) Nerve transfer

The most appropriate management for restoration of intrinsic muscle function is nerve transfer.In nerve injuries resulting in complete transection of the nerve, wallerian degeneration occurs at the site of transection, and Schwann cells in the distal nerve segment undergo apoptosis. With prolonged denervation, decreased regenerative ability with limitation in motor recovery is noted. Optimal functional recovery is dependent upon adequate reinnervation of the motor end plates and target muscles by regenerating motor axons. Over time, loss of target motor end plates via degeneration and fibrosis and replacement of muscle fibers by fat cells occur.

Nerve regeneration occurs at a rate of approximately 1 mm daily or 1 inch monthly. In a high injury to the ulnar nerve, the distance from the proximal motor axons to the intrinsic musculature precludes timely reinnervation, and intrinsic recovery is generally poor. Reinnervation of the muscle ideally should be completed within 12 to 18 months following injury to allow for recovery

In the patient who has had the delayed symptoms and high ulnar nerve injury described, the time torecovery of intrinsic function will be greater than 2 years if the injury is reconstructed directly. This estimate is based on the elapsed time and distance to the target muscles.Nerve transfer involves the use of a noncritical or expendable donor motor nerve to reinnervate a missing function. The selection of an available motor nerve donor that is closer to the target muscle can decrease the time needed for reinnervation of the muscle and help to ensure recovery before irreversible changes occur. In the scenario described, the distal portion of the anterior interosseous nerve can be used as a donor nerve to reinnervate the ulnar motor branch. Transfer of the distal anterior interosseous nerve to the motor branch of the ulnar nerve will provide motor neurons in a more distal location to reinnervate the intrinsic muscles in the desired time frame.

How well did you know this?
1
Not at all
2
3
4
5
Perfectly
3
Q

Nerve regeneration occurs at what rate?

A

Nerve regeneration occurs at a rate of approximately 1 mm daily or 1 inch monthly.

How well did you know this?
1
Not at all
2
3
4
5
Perfectly
4
Q

High ulnar nerve injury and intrinsic recovery

A

In a high injury to the ulnar nerve, the distance from the proximal motor axons to the intrinsic musculature precludes timely reinnervation, and intrinsic recovery is generally poor.

How well did you know this?
1
Not at all
2
3
4
5
Perfectly
5
Q

In what time frame should muscle reinnervation be performed?

A

Reinnervation of the muscle ideally should be completed within 12 to 18 months following injury to allow for recovery

How well did you know this?
1
Not at all
2
3
4
5
Perfectly
6
Q

Typically, conduits are used for:

A

Typically, conduits are used for sensory nerves in noncritical areas.

How well did you know this?
1
Not at all
2
3
4
5
Perfectly
7
Q

Gaps of up to ______ can be bridged with nerve conduits

A

Gaps of up to 2 to 3 cm can be bridged.

How well did you know this?
1
Not at all
2
3
4
5
Perfectly
8
Q
A 13-year-old boy is brought to the office because he has difficulty opening his hand and extending his fingers. History includes release of the forearm compartments to treat a pulseless hand following a supracondylar humerus fracture 2 years ago. On physical examination, passive extension of the fingers is restricted when the wrist is fully extended; it improves with full wrist flexion. Which of the following muscles is the most likely cause of the limitation described?
A) Flexor carpi radialis
B) Flexor carpi ulnaris
C) Flexor digitorum profundus
D) Flexor digitorum superficialis
E) Lumbricals
A

C) Flexor digitorum profundus

The most likely cause of the restricted finger extension described is fibrosis of the flexor digitorum profundus muscle. The patient exhibits Volkmann ischemic contracture as a complication of late treatment (over 24 hours from the time of initial ischemia) of arterial compromise associated with the fracture. The muscle groups at the greatest risk during these ischemic episodes are within the deep flexor compartment of the forearm. This risk occurs because the arterial supply is relatively distant from the usual site of occlusion and because this compartment is relatively less distensible. In the scenario described, the flexor digitorum profundus and flexor pollicis longus are at the greatest risk. Superficial muscle groups such as the flexor carpi radialis, flexor carpi ulnaris, and the flexor digitorum superficialis typically recover some function and do not lead to contractures in the forearm. Likewise, the small muscles of the hand, such as the lumbricals, tend to be less severely injured than the deep compartment of the forearm.

How well did you know this?
1
Not at all
2
3
4
5
Perfectly
9
Q

What would qualify as late treatment for upper extremity ischemia?

A

Over 24 hours from the time of initial ischemia

How well did you know this?
1
Not at all
2
3
4
5
Perfectly
10
Q

Which muscle groups are at greatest risk during upper extremity ischemia?

A

The muscle groups at the greatest risk during these ischemic episodes are within the deep flexor compartment of the forearm.

Superficial muscle groups such as the flexor carpi radialis, flexor carpi ulnaris, and the flexor digitorum superficialis typically recover some function and do not lead to contractures in the forearm. Likewise, the small muscles of the hand, such as the lumbricals, tend to be less severely injured than the deep compartment of the forearm.

How well did you know this?
1
Not at all
2
3
4
5
Perfectly
11
Q

Why is the deep flexor compartment of the forearm most at risk during an ischemic episode?

A

The greater risk occurs because the arterial supply is relatively distant from the usual site of occlusion and because this compartment is relatively less distensible.

How well did you know this?
1
Not at all
2
3
4
5
Perfectly
12
Q

A 24-year-old man comes to the office because he says the ring and little fingers of his right hand “catch” when he puts his hand in his pocket and that he “pokes” himself in the eye when washing his face. History includes repair of a complete transection of the right ulnar nerve at the wrist 1 year ago. On physical examination, he is unable to extend the interphalangeal joints of the ring and little fingers when the metacarpophalangeal joints are flexed. Photographs are shown. Which of the following tendon transfers is the most appropriate management?
A) Extensor indicis proprius (EIP) to adductor
B) EIP to extensor digiti minimi
C) EIP to first dorsal interosseous
D) Flexor digitorum superficialis (FDS) of the little finger to A2 pulley
E) FDS of the little finger to lateral band

A

E) FDS of the little finger to lateral band

Of the tendon transfer choices offered, only the FDS transfer to the lateral band (of both the ring and little fingers) will correct the loss of interphalangeal joint extension described, thereby diminishing the tendency for the flexed/abducted finger to catch on pocket edges. The clinical scenario and photographs demonstrate failure of the intrinsic muscle function to return following a low ulnar nerve repair. The deformities demonstrated include ulnar clawing of the little finger primarily, abduction of the little finger (Wartenbergsign), hyperflexion of the interphalangeal joint of the thumb, and atrophy of the intrinsics (especially notable in the hypothenar eminence on the lateral view). Of these deformities, the patient is bothered primarily by the little finger deformity.

Correction of clawing can be achieved actively or passively. Patients who can extend the interphalangeal joints while hyperextension of the metacarpophalangeal joints is blocked (Bouvier test) can achieve correction of clawing with active or passive transfers. Active transfers attempt to re-create the normal function of the intrinsics by directing pull through the lateral bands. Passive transfers re-create the intrinsic function of metacarpophalangeal joint flexion (similar to externally blocking hyperextension) but do not extend the interphalangeal joints. EIP transfers are useful for correction of the lateral pinch functions of the intrinsic minus hand. An EIP transfer to the adductor tendon re-creates the thumb component of lateral pinch, while the EIP transfer to the first dorsal interosseous tendon would improve the index function in pinch.

EIP transfer to the extensor digiti minimi is one method used to reduce hyperabduction of the little finger. This would not correct the flexion deformity at the level of the proximal interphalangeal joint.FDS transfer to the A2 pulley provides a passive transfer, which, based on the patient’s inability to extend the interphalangeal joints during the Bouvier test, would not correct the deformity.

How well did you know this?
1
Not at all
2
3
4
5
Perfectly
13
Q

Active transfers for correcting ulnar clawing: General concept

A

Active transfers attempt to re-create the normal function of the intrinsics by directing pull through the lateral bands.

How well did you know this?
1
Not at all
2
3
4
5
Perfectly
14
Q

Passive transfers for correcting ulnar clawing: General concept

A

Passive transfers re-create the intrinsic function of metacarpophalangeal joint flexion (similar to externally blocking hyperextension) but do not extend the interphalangeal joints.

How well did you know this?
1
Not at all
2
3
4
5
Perfectly
15
Q

Bouvier test

A

Correction of clawing can be achieved actively or passively. Patients who can extend the interphalangeal joints while hyperextension of the metacarpophalangeal joints is blocked (Bouvier test) can achieve correction of clawing with active or passive transfers.

How well did you know this?
1
Not at all
2
3
4
5
Perfectly
16
Q
A 25-year-old right-hand dominant man is brought to the emergency department after sustaining a stab wound to the right arm in a bar fight. Physical examination shows a 2 × 1-cm laceration over the antecubital fossa. He is unable to flex the interphalangeal joint of the thumb and the proximal interphalangeal joint of the index finger. Which of the following nerves is most likely injured?
A) Lateral antebrachial cutaneous
B) Median
C) Musculocutaneous
D) Radial
E) Ulnar
A

B) Median

Median nerve palsy is marked by the inability to oppose the thumb or flex the thumb at the interphalangeal joint. The inability to flex the index finger at the proximal interphalangeal joint is also noted. The lateral antebrachial cutaneous nerve provides sensory innervation to the lateral aspect of the arm. The median antebrachial cutaneous nerve innervates the skin of the anterior and middle surfaces of the forearm to the level of the wrist. This nerve does not innervate any muscles. Radial nerve palsy is marked by the inability to extend the fingers, thumb, and wrist. Patients with radial nerve palsies have difficulty grasping objects. The results of tendon transfers to restore function in patients with radial nerve palsies are among the best and most predictable outcomes. Ulnar nerve palsy symptoms include a “claw” deformity, with flexion deformities of the ring and little fingers. In later stages, profound muscle wasting of the both hypothenar eminence and the first web space is seen.

How well did you know this?
1
Not at all
2
3
4
5
Perfectly
17
Q

Symptoms of median nerve palsy

A

Median nerve palsy is marked by the inability to oppose the thumb or flex the thumb at the interphalangeal joint. The inability to flex the index finger at the proximal interphalangeal joint is also noted.

How well did you know this?
1
Not at all
2
3
4
5
Perfectly
18
Q

The lateral antebrachial cutaneous nerve innervates what?

A

The lateral antebrachial cutaneous nerve provides sensory innervation to the lateral aspect of the arm.

How well did you know this?
1
Not at all
2
3
4
5
Perfectly
19
Q

The median antebrachial cutaneous nerve innervates what?

A

The median antebrachial cutaneous nerve innervates the skin of the anterior and middle surfaces of the forearm to the level of the wrist. This nerve does not innervate any muscles.

How well did you know this?
1
Not at all
2
3
4
5
Perfectly
20
Q

Symptoms of radial nerve palsy

A

Radial nerve palsy is marked by the inability to extend the fingers, thumb, and wrist. Patients with radial nerve palsies have difficulty grasping objects.

How well did you know this?
1
Not at all
2
3
4
5
Perfectly
21
Q

Symptoms of ulnar nerve palsy

A

Ulnar nerve palsy symptoms include a “claw” deformity, with flexion deformities of the ring and little fingers. In later stages, profound muscle wasting of the both hypothenar eminence and the first web space is seen.

How well did you know this?
1
Not at all
2
3
4
5
Perfectly
22
Q

A 55-year-old man with bilateral carpal tunnel syndrome comes to the outpatient surgical unit for elective surgical intervention of the dominant right hand. He will be the tenth procedure of the day for the surgeon performing the operation. The surgeon favors an open technique; he has performed 150 carpal tunnel operations since finishing his hand fellowship 3 years ago. Which of the following is most likely to increase the risk of wrong-site surgery?
A) The elective nature of the procedure
B) Only one surgeon is involved in the operation
C) The procedure will be performed using an open technique
D) The surgeon has a high volume of cases scheduled for the same day
E) The surgeon has only been practicing independently for 3 years

A

D) The surgeon has a high volume of cases scheduled for the same day

A Joint Commission review of a series of sentinel events identified a number of factors contributing to the increased risk of wrong-site surgery, such as emergency cases; unusual physical characteristics, including morbid obesity or physical deformity; unusual time pressures to start or complete the procedure; unusual equipment or setup in the operating room; multiple surgeons involved in the case; and multiple procedures being performed during a single surgical visit. A large series of wrong-site hand surgeries showed an increased rate of wrong-site surgery with increasing surgeon age and experience, and a direct correlation with increasing surgical case volumes.

How well did you know this?
1
Not at all
2
3
4
5
Perfectly
23
Q
A 34-year-old man is brought to the emergency department 2 hours after sustaining injuries to the right wrist when he punched a glass window. Surgical exploration shows a complete laceration of the median nerve at the level of the wrist. A 1-cm gap between the proximal and the distal stumps of the nerve is noted. Which of the following treatments is most likely to provide the best functional outcome?
A) Multistrand nerve grafting
B) Nerve transfer
C) Nerve transposition
D) Primary epineurial repair
E) Single-strand nerve grafting
A

D) Primary epineurial repair

The need for nerve grafting is dependent upon many parameters, such as the length of the gap, the excursion of the nerve, the wound bed, and vascularity, among others. For clean, sharp injuries with nerve gaps measuring less than 1 cm in a large peripheral nerve such as the median, most authors agree that primary repair of the nerve results in the best outcome. Autologous nerve grafting should be reserved for cases in which there is tension on the nerve ends with primary repair. Both multistrand nerve grafting and single-strand nerve grafting produce similar outcomes and are inferior to primary repair. Nerve transfer would only be considered if there were no proximal nerve to repair to the distal nerve. Nerve transposition would only be appropriate for gaining length in the ulnar nerve, where the switch from the extensor side of the elbow to the flexor side results in increased relative length in the nerve

How well did you know this?
1
Not at all
2
3
4
5
Perfectly
24
Q

When should primary nerve repair be performed in the upper extremity?

A

For clean, sharp injuries with nerve gaps measuring less than 1 cm in a large peripheral nerve such as the median, most authors agree that primary repair of the nerve results in the best outcome. (Acute setting)

How well did you know this?
1
Not at all
2
3
4
5
Perfectly
25
Q

When should autologous nerve grafting be performed, instead of primary nerve repair?

A

Autologous nerve grafting should be reserved for cases in which there is tension on the nerve ends with primary repair.

How well did you know this?
1
Not at all
2
3
4
5
Perfectly
26
Q

Multistrand nerve grafting versus single strand nerve grafting

A

Both multistrand nerve grafting and single-strand nerve grafting produce similar outcomes and are inferior to primary repair

How well did you know this?
1
Not at all
2
3
4
5
Perfectly
27
Q

A 40-year-old man who is an avid cyclist comes for evaluation of a 5-week history of numbness and tingling of the ring and little fingers of the left hand. Conservative management has failed. Physical examination shows weakness of grip strength. Sensation over the dorsal ulnar aspect of the hand is intact, and results of elbow flexion testing are negative. Which of the following structures are most appropriate for surgical release?
A ) Osborne ligament and arcade of Struthers
B ) Osborne ligament and volar carpal ligament
C ) Pisohamate ligament and volar carpal ligament
D ) Transverse carpal ligament and arcade of Struthers
E ) Transverse carpal ligament and pisohamate ligament

A

C ) Pisohamate ligament and volar carpal ligament

The patient described has ulnar tunnel syndrome or compression of the ulnar nerve in the Guyon canal. Sensory sparing onthe dorsal ulnar aspect of the hand suggests a lesion distal to the origin of the dorsal cutaneous branch of the ulnar nerve. Nerve conduction studies and electromyography can be used to confirm the diagnosis. Conservative treatment includes activity modification, splinting, and administration of a nonsteroidal anti-inflammatory drug.
The most appropriate management of this condition is exploration of the entire ulnar tunnel and release of the pisohamate and volar carpal ligaments. The ulnar tunnel, whichis 4 to 4.5 cm in length, begins at the proximal volar carpal ligament and ends at the fibrous edge of the hypothenar muscles. Zone I is the region of the tunnel proximal to the bifurcation of the nerve. Zone II is the area around the deep motor branch, which ends at the pisohamate ligament, and Zone III is the area surrounding the superficial branch. The ulnar nerve courses between the volar carpal ligament and the transverse carpal ligament.
The Osborne ligament and the arcade of Struthers are potentialsites of compression of the ulnar nerve at the elbow, or cubital tunnel syndrome. The fascia of Osborne, or the cubital tunnel retinaculum, is a band bridging the two origins of the flexor carpi ulnaris muscle and the medial epicondyle. The arcade of Struthers is 8 to 10 cm proximal to the medial epicondyle and extends from the medial intermuscular septum to the medial head of the triceps.
Release of the transverse carpal ligament is appropriate for management of compression of the median nerve at the wrist, or carpal tunnel syndrome.

How well did you know this?
1
Not at all
2
3
4
5
Perfectly
28
Q

Conservative management of Ulnar tunnel compression

A

Conservative treatment includes activity modification, splinting, and administration of a nonsteroidal anti-inflammatory drug.

How well did you know this?
1
Not at all
2
3
4
5
Perfectly
29
Q

Ulnar tunnel syndrome: What suggests compression distal to the dorsal cutaneous branch?

A

Sensory sparing onthe dorsal ulnar aspect of the hand suggests a lesion distal to the origin of the dorsal cutaneous branch of the ulnar nerve.

How well did you know this?
1
Not at all
2
3
4
5
Perfectly
30
Q

Management of ulnar tunnel syndrome

A

The most appropriate management of this condition is exploration of the entire ulnar tunnel and release of the pisohamate and volar carpal ligaments.

How well did you know this?
1
Not at all
2
3
4
5
Perfectly
31
Q

Ulnar tunnel: Anatomy

A

The ulnar tunnel, whichis 4 to 4.5 cm in length, begins at the proximal volar carpal ligament and ends at the fibrous edge of the hypothenar muscles.
The ulnar nerve courses between the volar carpal ligament and the transverse carpal ligament.

How well did you know this?
1
Not at all
2
3
4
5
Perfectly
32
Q

Ulnar tunnel: Zone I

A

Zone I is the region of the tunnel proximal to the bifurcation of the nerve.

How well did you know this?
1
Not at all
2
3
4
5
Perfectly
33
Q

Ulnar tunnel: Zone II

A

Zone II is the area around the deep motor branch, which ends at the pisohamate ligament

How well did you know this?
1
Not at all
2
3
4
5
Perfectly
34
Q

Ulnar tunnel: Zone III

A

Zone III is the area surrounding the superficial branch.

How well did you know this?
1
Not at all
2
3
4
5
Perfectly
35
Q

The fascia of Osborne

A

The fascia of Osborne, or the cubital tunnel retinaculum, is a band bridging the two origins of the flexor carpi ulnaris muscle and the medial epicondyle.

How well did you know this?
1
Not at all
2
3
4
5
Perfectly
36
Q

Potential sites of compression of the ulnar nerve at the elbow:

A

The Osborne ligament and the arcade of Struthers are potential sites of compression of the ulnar nerve at the elbow, or cubital tunnel syndrome.

How well did you know this?
1
Not at all
2
3
4
5
Perfectly
37
Q

The arcade of Struthers

A

The arcade of Struthers is 8 to 10 cm proximal to the medial epicondyle and extends from the medial intermuscular septum to the medial head of the triceps.

How well did you know this?
1
Not at all
2
3
4
5
Perfectly
38
Q
A 45-year-old man is brought to the emergency department immediately after sustaining injuries to the right upper extremity during a motorcycle collision. On admission, physical examination shows a flaccid and insensate right upper extremity, and x-ray studies show no abnormalities. Follow-up nerve conduction studies 4 weeks later show preservation of sensory nerve action potentials and evidence of fibrillations and denervation in the motor action potentials from the paravertebral muscles, biceps, triceps, and deltoid. The nerve injury is most likely located at which of the following levels of the nerve?
A ) Anterior division
B ) C5 and C6 trunk
C ) Lateral cord
D ) Preganglionic root
E ) Subscapular nerve
A

D ) Preganglionic root

Following a traction injury to the brachial plexus, the nerves may rupture, be avulsed at the level of the spinal cord, or significantly stretch but remain intact. There are five possible levels where the nerve can be injured: (1) root, (2) anterior branches of the spinal nerves, (3) trunk, (4) cord, and (5) peripheral nerve. Root injuries may be further localized with respect to the dorsal root ganglion. Infraganglionic (postganglionic) injuries are located distal to the dorsal root ganglion, and supraganglionic (preganglionic) lesions are located proximal to the ganglion. With both types of lesions, patients will have the symptom of loss of muscle function. In supraganglionic injuries, the nerve has been avulsed from the spinal cord, separating the motor nerve fibers from the motor cell bodies in the anterior horn cells. The sensory fibers and cell bodies are still connected at the dorsal root ganglion; however, the efferent fibers entering the dorsal spinal column have been disrupted. Thus, sensory nerve action potentials will be preserved in patients with supraganglionic injuries, while motor nerve action potentials will be absent. In infraganglionic injuries, both the motor and sensory nerve cells have been disrupted, so there will be abnormalities in both motor and sensory action potentials. An injury to the C5 and C6 roots would show preservation of triceps function, as would an injury to the medial or lateral cord.

How well did you know this?
1
Not at all
2
3
4
5
Perfectly
39
Q

Possible levels of injury of the brachial plexus

A

There are five possible levels where the nerve can be injured: (1) root, (2) anterior branches of the spinal nerves, (3) trunk, (4) cord, and (5) peripheral nerve.

How well did you know this?
1
Not at all
2
3
4
5
Perfectly
40
Q

Sub levels of brachial plexus root injuries

A

Root injuries may be further localized with respect to the dorsal root ganglion:

  • Infraganglionic (postganglionic) injuries are located distal to the dorsal root ganglion
  • Supraganglionic (preganglionic) lesions are located proximal to the ganglion.
How well did you know this?
1
Not at all
2
3
4
5
Perfectly
41
Q

Supraganglionic brachial plexus injuries

A

In supraganglionic injuries, the nerve has been avulsed from the spinal cord, separating the motor nerve fibers from the motor cell bodies in the anterior horn cells. The sensory fibers and cell bodies are still connected at the dorsal root ganglion; however, the efferent fibers entering the dorsal spinal column have been disrupted. Thus, sensory nerve action potentials will be preserved in patients with supraganglionic injuries, while motor nerve action potentials will be absent.

How well did you know this?
1
Not at all
2
3
4
5
Perfectly
42
Q

Infraganglionic brachial plexus injuries

A

In infraganglionic injuries, both the motor and sensory nerve cells have been disrupted, so there will be abnormalities in both motor and sensory action potentials.

How well did you know this?
1
Not at all
2
3
4
5
Perfectly
43
Q

A 48-year-old woman has numbness and paresthesia of the right hand 2 years after mastectomy and radiation therapy. She reports no pain or night waking. Symptoms have not improved with cock-up wrist splints or injection of a corticosteroid into the carpal tunnel. Physical examination shows swelling and weakness of the right arm, most prominently in the C5-C6 distribution; no varicosities, stasis ulcers, dermatitis, or symptoms of Horner syndrome are noted. Allen test is normal. Electromyography shows myokymia. CT scan shows diffuse swelling but no mass. Which of the following is the most likely diagnosis?
A ) Acute ischemic injury
B ) Carpal tunnel syndrome
C ) Chronic venous insufficiency
D ) Radiation-induced brachial plexopathy
E ) Tumor recurrence

A

D ) Radiation-induced brachial plexopathy

The most likely diagnosis is radiation-induced brachial plexopathy, which can occur when radiation therapy is directed at the chest, axillary region, thoracic outlet, or neck. The incidence is 1.8 to 4.9% of those patients receiving radiation therapy to the above areas and is most common in patients with underlying breast or lung carcinoma. Patients often have sensory symptoms, with swelling and a generalized weakness of the arm.
Eighteen percent of patients have pain in the shoulder, wrist, or hand. The neurologic findings are most prominent in the C5-C6 distribution. The lymphatic-vascular system may show prominent lymphedema of the involved extremity without cyanotic or dusky features. There should be no disturbance of arterial or venous circulation in the involved extremity and no changes in the limb to suggest venous insufficiency (ie, varicosities, stasis ulcers, or dermatitis). The Allen test should be normal. Horner syndrome is not present in patients with radiation-induced brachial plexopathy.Eighty percent of patients with tumor infiltration into the brachial plexus come to the office because of pain in the shoulder, upper arm, elbow, and ring and little fingers. Symptoms progress to atrophy and weakness of the C7-T1 distribution with persistent pain and occasional Horner syndrome. CT scan shows a discrete mass with circumscribed tissue infiltration. Electromyography shows segmental slowing.
Patients with acute ischemic injury have symptoms of paresthesia in the C5-C6 nerve distribution and acute, nonprogressive weakness and sensory loss. CT angiography woulddemonstrate subclavian artery segmental obstruction. Electromyography shows segmental slowing.
Patients with carpal tunnel syndrome often have night waking and experience a period of symptomatic relief after injection of a corticosteroid into the carpal tunnel.
Chronic venous insufficiency does not typically have neurologic sequela

How well did you know this?
1
Not at all
2
3
4
5
Perfectly
44
Q

When does radiation induced brachial plexopathy occur?

In what % of patients?

A

Can occur when radiation therapy is directed at the chest, axillary region, thoracic outlet, or neck. The incidence is 1.8 to 4.9% of those patients receiving radiation therapy to the above areas

How well did you know this?
1
Not at all
2
3
4
5
Perfectly
45
Q

Presentation of radiation induced brachial plexopathy

A
  • Sensory symptoms, with swelling and a generalized weakness of the arm.
  • 19% of patients have pain in the shoulder, wrist, or hand.
  • The neurologic findings are most prominent in the C5-C6 distribution.

The lymphatic-vascular system may show prominent lymphedema of the involved extremity without cyanotic or dusky features. There should be no disturbance of arterial or venous circulation in the involved extremity and no changes in the limb to suggest venous insufficiency (ie, varicosities, stasis ulcers, or dermatitis). The Allen test should be normal.

How well did you know this?
1
Not at all
2
3
4
5
Perfectly
46
Q

Presentation of patient with tumor infiltration into the brachial plexus

A

80% of patients with tumor infiltration into the brachial plexus come to the office because of pain in the shoulder, upper arm, elbow, and ring and little fingers. Symptoms progress to atrophy and weakness of the C7-T1 distribution with persistent pain and occasional Horner syndrome.

How well did you know this?
1
Not at all
2
3
4
5
Perfectly
47
Q
A 54-year-old woman comes to the office because of a 6-year history of weakness and numbness of the left hand. Physical examination shows decreased sensation in the thumb, index, long, and ring fingers. No other sensory abnormalities are noted. Examination of which of the following muscles is most likely to confirm a diagnosis?
A ) Abductor pollicis brevis
B ) Adductor pollicis
C ) First dorsal interosseous
D ) Flexor digiti minimi
E ) Flexor pollicis brevis
A

A ) Abductor pollicis brevis

The examination of the patient described suggests an injury or compression neuropathy of the median nerve.

The only intrinsic muscle innervated by the median nerve (recurrent branch) that can be reliably tested separately from the ulnar intrinsic muscles is the abductor pollicis brevis. The adductor pollicis, first dorsal interosseous, and flexor digiti minimiare all completely innervated by the ulnar nerve. The flexor pollicis brevis muscle has dual innervation from both the ulnar (deep head) and median (superficial head) nerves.

How well did you know this?
1
Not at all
2
3
4
5
Perfectly
48
Q

What is the only intrinsic muscle that can be tested independently of the ulnar intrinsic muscles

A

The only intrinsic muscle innervated by the median nerve (recurrent branch) that can be reliably tested separately from the ulnar intrinsic muscles is the abductor pollicis brevis.

How well did you know this?
1
Not at all
2
3
4
5
Perfectly
49
Q

Innervation of the flexor pollicis brevis muscle

A

The flexor pollicis brevis muscle has dual innervation from both the ulnar (deep head) and median (superficial head) nerves

How well did you know this?
1
Not at all
2
3
4
5
Perfectly
50
Q
A 30-year-old woman comes to the emergency department after cutting herself with a kitchen knife. Surgical exploration shows that the median nerve had been cut at the distal forearm. The distal nerve is relatively fixed, but the proximal nerve has been lifted and twisted. Immediate repair is planned. The motor fibers of the proximal nerve end are most likely in which of the following locations relative to the sensory fibers?
A ) Dorsal and radial
B ) Dorsal and ulnar
C ) Volar and radial
D ) Volar and ulnar
A

C ) Volar and radial

At the level of the distal forearm, the median nerve is a mixed nerve comprised of both motor (20%) and sensory (80%) fibers. The motor fibers become the thenar branch, which innervates the abductor pollicis brevis (AbPB), opponens pollicis (OP), and flexor pollicis brevis (FPB). The FPB is located distal to the AbPB and OP and also has innervation from the ulnar nerve, which is why patients with median nerve injuries can sometimes still bring the thumb to the little finger. The thenar nerve can be injured with acarpal tunnel release procedure. After coursing into the carpal tunnel, these median nerve motor fibers leave the median nerve volar and radial to the sensory fibers of the median nerve through a variety of branching patterns.

When patients have clean-cut injuries to nerves, the nerves can be repaired primarily. In a contaminated, dirty wound caused by a crush and mutilating injury, it can be difficult to determine the nonviable nerve required for debridement before repair. In the scenario described, the nerve ends are labeled with a permanent suture for later identification and repair at a second stage. In general, sensory-only nerves can be repaired by epineurial approximation, and group fascicular repair can be considered for mixed nerves.

How well did you know this?
1
Not at all
2
3
4
5
Perfectly
51
Q

At the level of the distal forearm, the median nerve is a ______ nerve comprised of:

A

At the level of the distal forearm, the median nerve is a mixed nerve comprised of both motor (20%) and sensory (80%) fibers.

How well did you know this?
1
Not at all
2
3
4
5
Perfectly
52
Q

The ______ branch of the median nerve innervates what muscles?

A

The motor fibers of the thenar nerve become the thenar branch, which innervates the:

  • abductor pollicis brevis (AbPB)
  • opponens pollicis (OP)
  • flexor pollicis brevis (FPB). The FPB is located distal to the AbPB and OP and also has innervation from the ulnar nerve, which is why patients with median nerve injuries can sometimes still bring the thumb to the little finger.
How well did you know this?
1
Not at all
2
3
4
5
Perfectly
53
Q

Why would a patient with a median nerve injury sometimes be able to bring the thumb to the little finger?

A

The FPB is located distal to the AbPB and OP and also has innervation from the ulnar nerve, which is why patients with median nerve injuries can sometimes still bring the thumb to the little finger.

How well did you know this?
1
Not at all
2
3
4
5
Perfectly
54
Q

After coursing into the carpal tunnel, the course of the median nerve motor fibers, relative to the sensory fibers:

A

After coursing into the carpal tunnel, these median nerve motor fibers leave the median nerve volar and radial to the sensory fibers of the median nerve through a variety of branching patterns.

How well did you know this?
1
Not at all
2
3
4
5
Perfectly
55
Q

A 68-year-old woman comes to the office because of a 4-year history of numbness and tingling in the tips of the thumb, index, and long fingers. The symptoms have become progressively severe and now wake her at night. Physical examination shows decreased strength of thumb opposition and thenar atrophy. Sensation to pinprick is diminished over the median nerve distribution. Which of the following findings is most likely on electrodiagnostic testing?
Median sensory latency, Median motor latency, Fibrillations
A ) Decreased || decreased || absent
B ) Decreased || decreased || present
C ) Unchanged || unchanged || absent
D ) Increased || increased || absent
E ) Increased || increased || present

A

E ) Increased || increased || present

The most likely finding on electrodiagnostic testing is increased median sensory latency, increased median motor latency, and presence of fibrillations.
For the patient described, compression of the median nerve results in numbness and paresthesias in the median nerve distribution. The disease has progressed to the point where physical findings include decreased strength and thenar atrophy, indicating the presence of denervation and median nerve damage.
On nerve conduction studies (NCS), the sensory and motor latency measurements detect the time necessary for a signal to travel across a segment of nerve from a stimulating electrode to a recording electrode. In the presence ofnerve compression, demyelination results and conduction is slowed, resulting in a longer time and increased latency (measured in milliseconds).
On electromyography (EMG), the needle examination will assess the presence of denervation changes, manifested as fibrillations and positive sharp waves. These are signs of ongoing nerve injury and muscle denervation. Spontaneous muscle activity with fibrillation potentials is the earliest sign of denervation. In severe carpal tunnel syndrome, examination of the median-innervated abductor pollicis brevis muscle will reveal changes. There may also be alteration in the motor unit potentials caused by chronic episodes of denervation and reinnervation.Carpal tunnel syndrome remains a clinical diagnosis based on historyand physical examination. Clinical suggestion may be confirmed by electrophysiologic testing, but false-negative and false-positive results may occur.
How well did you know this?
1
Not at all
2
3
4
5
Perfectly
56
Q

On nerve conduction studies, the latency measurements detect what?

A

On nerve conduction studies (NCS), the sensory and motor latency measurements detect the time necessary for a signal to travel across a segment of nerve from a stimulating electrode to a recording electrode.

How well did you know this?
1
Not at all
2
3
4
5
Perfectly
57
Q

EMG needle examination:

A

On electromyography (EMG), the needle examination will assess the presence of denervation changes, manifested as fibrillations and positive sharp waves. These are signs of ongoing nerve injury and muscle denervation.

How well did you know this?
1
Not at all
2
3
4
5
Perfectly
58
Q

EMG: Earliest sign of denervation

A

Spontaneous muscle activity with fibrillation potentials is the earliest sign of denervation.

How well did you know this?
1
Not at all
2
3
4
5
Perfectly
59
Q
Volkmann ischemic contracture occurs when forearm compartment pressure is above 30 mmHg for a minimum of approximately how many hours?
A ) Less than 2
B ) 3-5
C ) 6-12
D ) 18-24
E ) Greater than 24
A

C ) 6-12

Permanent neuromuscular damage occurs at 12 hours, leading to subsequent Volkmann contracture. Timeline of ischemia, shown experimentally, is capillary endothelial damage at 3 hours; partially reversible muscle and nerve injury occurs at 6 hours.

How well did you know this?
1
Not at all
2
3
4
5
Perfectly
60
Q

Upper extremity ischemia: Permanent neuromuscular damage occurs when?

A

Permanent neuromuscular damage occurs at 12 hours, leading to subsequent Volkmann contracture.

How well did you know this?
1
Not at all
2
3
4
5
Perfectly
61
Q

Upper extremity ischemia: Timeline

A

Timeline of ischemia, shown experimentally, is:

3 hours: capillary endothelial damage
6 hours: partially reversible muscle and nerve injury

How well did you know this?
1
Not at all
2
3
4
5
Perfectly
62
Q

A 30-year-old man who works as an ironworker is brought to the emergency department after falling 20 ft and impaling his right upper arm on a picket fence. Surgical exploration shows a complete transection of the ulnar nerve 6 in (15.2 cm) above the elbow with a 2-cm nerve gap. The median and radial nerves are intact. Which of the following procedures is most likely to result in maximal recovery of intrinsic muscle function?
A ) Anterior transposition of the ulnar nerve at the elbow followed by primary repair
B ) Interposition nerve grafting using the lateral antebrachial cutaneous nerve
C ) Nerve transfer of the distal anterior interosseous nerve to the deep motor branch of the ulnar nerve
D ) Tendon transfer of the extensor digiti quinti to the extensor digitorum communis
E ) Vascularized radial nerve grafting

A

C ) Nerve transfer of the distal anterior interosseous nerve to the deep motor branch of the ulnar nerve

High ulnar nerve injuries, ie, above the elbow, have routinely yielded unsatisfactory results with minimal return of intrinsic function and resultant claw hand deformity. This poor recovery of motor function of the intrinsic muscles of the hand occurs despite meticulous neurorrhaphy technique and regardless of whether nerve graft or primary repair is performed. The principal reason for poor results is the long distance between the site of injury and the muscle motor endplates. During the many months required for the regenerating axons to traverse this gap, the target muscles are undergoing atrophy and fibrosis. The rate of nerve regeneration is approximately 1 inch per day. Motor endplates become refractory to reinnervation after about 15 to 18 months. In contrast, sensory return from high nerve injuries is usually adequate.

The key to nerve transfer is to convert a high nerve injury to a low nerve injury. In this case, the original injury is over 15 inches from the ulnar nerve innervations of the intrinsic muscles. The likelihood of any intrinsic muscle function is poor. By transferring the distal anterior interosseous nerve at the level of the pronator quadratus to the deep motor branch of the ulnar nerve, the distance that the reinnervating nerve has to travel is much shorter, thus preserving the muscle structure.

The other options presented are all inappropriate because they do not address the length of time it will take the nerve to grow to the motor endplates.

How well did you know this?
1
Not at all
2
3
4
5
Perfectly
63
Q

High ulnar nerve injuries: Reason for poor results?

A

The principal reason for poor results is the long distance between the site of injury and the muscle motor endplates. During the many months required for the regenerating axons to traverse this gap, the target muscles are undergoing atrophy and fibrosis.

How well did you know this?
1
Not at all
2
3
4
5
Perfectly
64
Q

Motor endplates become refractory to reinnervation after how long?

A

Motor endplates become refractory to reinnervation after about 15 to 18 months.

How well did you know this?
1
Not at all
2
3
4
5
Perfectly
65
Q

Sensory return from high nerve injuries

A

Sensory return from high nerve injuries is usually adequate.

How well did you know this?
1
Not at all
2
3
4
5
Perfectly
66
Q

The key to nerve transfer is what?

A

The key to nerve transfer is to convert a high nerve injury to a low nerve injury.

How well did you know this?
1
Not at all
2
3
4
5
Perfectly
67
Q

Morbidity of transferring the distal anterior interosseous nerve at the level of the pronator quadratus:

A

The loss of pronator quadratus function is insignificant.

How well did you know this?
1
Not at all
2
3
4
5
Perfectly
68
Q

Determining the need for nerve transfer:

A

When determining the need for nerve transfer, the time vs. distance constraints can be guided by the “Rule of 18.” The Rule of 18 states: “The number of inches from the site of nerve injury to the supplied muscle added to the number of months the muscle has been denervated should be less than the number 18 in order for the primary nerve reconstruction to be attempted successfully.” In other words, the nerve must be able to get to the muscle before 18 months from the initial injury.

How well did you know this?
1
Not at all
2
3
4
5
Perfectly
69
Q

Rule of 18:

A

When determining the need for nerve transfer, the time vs. distance constraints can be guided by the “Rule of 18.” The Rule of 18 states: “The number of inches from the site of nerve injury to the supplied muscle added to the number of months the muscle has been denervated should be less than the number 18 in order for the primary nerve reconstruction to be attempted successfully.” In other words, the nerve must be able to get to the muscle before 18 months from the initial injury.

How well did you know this?
1
Not at all
2
3
4
5
Perfectly
70
Q
A newborn who sustained a traction injury to the shoulder during delivery because of the use of forceps has complete palsy of the left upper extremity. Surgical intervention is indicated if there is no spontaneous biceps recovery by what age?
A ) 6 weeks
B ) 6 months
C ) 18 months
D ) 24 months
E ) 36 months
A

B ) 6 months

Most experts will perform surgery on infants who are between 3 and 6 months of age. At 3 months of age, if biceps function is absent, there is a poor prognosis. Early on, 92% will recover spontaneously. If biceps function is noted, surgery is not recommended. At 3 months of age, the child is retested by electromyography if there is no biceps function, although there is poor correlation with final results. The final evaluation is cervical myelography.

How well did you know this?
1
Not at all
2
3
4
5
Perfectly
71
Q

Traction injury in a newborn: At what point is the prognosis poor

A

At 3 months of age, if biceps function is absent, there is a poor prognosis.

How well did you know this?
1
Not at all
2
3
4
5
Perfectly
72
Q

___% newborns with traction injury recover spontaneously <3 months of age

A

Early on, 92% will recover spontaneously.

How well did you know this?
1
Not at all
2
3
4
5
Perfectly
73
Q

Age range appropriate for surgery for newborn traction injury

A

Most experts will perform surgery on infants who are between 3 and 6 months of age.

How well did you know this?
1
Not at all
2
3
4
5
Perfectly
74
Q

A 23-year-old man is brought to the emergency department after sustaining a single gunshot wound to the right upper arm. Physical examination shows a high radial nerve palsy. Which of the following is the most appropriate first step in management of potential nerve injury?
A ) Immediate surgical exploration and primary nerve repair if nerve is lacerated
B ) Immediate surgical exploration and repair with a nerve graft if nerve is lacerated
C ) Immediate surgical exploration, resection of devitalized nerve, and suture tagging of nerve ends for delayed repair
D ) Observation with electromyography six weeks after injury followed by exploration and repair if no return of function
E ) Observation with electromyography six months after injury followed by exploration and repair if no return of function

A

D ) Observation with electromyography six weeks after injury followed by exploration and repair if no return of function

In general, nerve injuries associated with open wounds should be explored and repaired early. If the nerve injury resulted from a relatively clean laceration or wound, it should be explored and repaired immediately. Crush or significant soft-tissue injury prohibits early nerve repair until the extent of devitalized tissue can be determined and the soft-tissue repair is stable.

Although gunshot wounds are technically open injuries, they should be treated as closed or blunt nervetrauma because the etiology of the trauma is predominately heat and shock. The majority of nerve dysfunction spontaneously recovers after gunshot wounds. Thus, the most logical first step in treatment is observation for at least six weeks at which time electrodiagnostic studies can be undertaken.

If there is not complete clinical return of function within six weeks, electrodiagnostic studies should be performed for baseline function and repeated at 12 weeks if clinical return of function is still not complete. If electromyography does demonstrate motor unit potentials, expectant management should be continued as full function should return. Electrodiagnostic studies should not be delayed for six months because long-term function is significantly decreased if nerve repair is delayed longer than three months.

Lack of clinical or electrical evidence of reinnervation at three months requires surgical exploration. A nerve that has been completely divided or fails to conduct intraoperatively should be managed withresection of scar tissue and repair of the nerve gap with an interpositional nerve graft. An in-continuity lesion should undergo neurolysis of nonfunctioning nerve units and nerve grafting. The functioning units can remain intact

How well did you know this?
1
Not at all
2
3
4
5
Perfectly
75
Q

When should a nerve be repaired immediately versus delayed?

A

If the nerve injury resulted from a relatively clean laceration or wound, it should be explored and repaired immediately. Crush or significant soft-tissue injury prohibits early nerve repair until the extent of devitalized tissue can be determined and the soft-tissue repair is stable.

How well did you know this?
1
Not at all
2
3
4
5
Perfectly
76
Q

Gunshot wounds should be treated as _______ wounds re: nerve trauma, because:

A

Although gunshot wounds are technically open injuries, they should be treated as closed or blunt nervetrauma because the etiology of the trauma is predominately heat and shock.

How well did you know this?
1
Not at all
2
3
4
5
Perfectly
77
Q

Management of nerve injury after gunshot injury

A

The majority of nerve dysfunction spontaneously recovers after gunshot wounds. Thus, the most logical first step in treatment is observation for at least six weeks at which time electrodiagnostic studies can be undertaken.

How well did you know this?
1
Not at all
2
3
4
5
Perfectly
78
Q

Nerve injury after gunshot wound: When should these be managed expectantly?

A

If there is not complete clinical return of function within six weeks, electrodiagnostic studies should be performed for baseline function and repeated at 12 weeks if clinical return of function is still not complete. If electromyography does demonstrate motor unit potentials, expectant management should be continued as full function should return.

How well did you know this?
1
Not at all
2
3
4
5
Perfectly
79
Q

Nerve injury after gunshot wound: When should these be managed surgically?

A

Lack of clinical or electrical evidence of reinnervation at three months requires surgical exploration.

How well did you know this?
1
Not at all
2
3
4
5
Perfectly
80
Q

Operative approach to nerve injury following a gunshot wound

A

A nerve that has been completely divided or fails to conduct intraoperatively should be managed with resection of scar tissue and repair of the nerve gap with an interpositional nerve graft. An in-continuity lesion should undergo neurolysis of nonfunctioning nerve units and nerve grafting. The functioning units can remain intact.

How well did you know this?
1
Not at all
2
3
4
5
Perfectly
81
Q

A 52-year-old man comes to the office because he has had a clinical diagnosis of carpal tunnel syndrome including numbness and dysesthesia of the thumb, index, and long fingers of both hands for the past four months. He says his symptoms worsen at night and when his wrist position changes while driving. Electrodiagnostic studies show delayed motor and sensory latencies and slowed conduction velocity at the wrist, indicating a conduction block of the median nerve. This block is most likely caused by localized nerve ischemia that results in which of the following?
A ) Blocking of calcium channels
B ) Failure to maintain the resting nerve potential at −30 mV
C ) Opening of sodium channels
D ) Prevention of nerve membrane depolarization
E ) Thickening of the myelin sheath

A

D ) Prevention of nerve membrane depolarization

Local nerve ischemia prevents depolarization. A charge is maintained across the axon membrane with the interior of the axon having a charge of −90 mV. When the resting membrane potential reaches −50 mV, the membrane depolarizes. Since there is a relatively greater concentration of K+ ions on the inside of the axon, there is a slow leak of K+ ions, which causes the inside of the axon to become less negative. The ATP-dependent pump imports K+ and exports Na+, maintaining the normal resting membrane potential at −90 mV. Maintenance of this ionic charge separation across the membrane requires energy, and the mechanism stops when the energy supply is interrupted, as with local ischemia. This is one of the mechanisms for conduction block that occurs with nerve compression. Various membrane channels consist of proteins embedded in the phospholipid membrane bilayer that allow passage of Na+, Ca2+, and K+ ions. With chronic nerve compression, the myelin sheath disintegrates and saltatory conduction between nodes of Ranvier ceases.

How well did you know this?
1
Not at all
2
3
4
5
Perfectly
82
Q

Local nerve ischemia prevents ____________.

A

Local nerve ischemia prevents depolarization.

How well did you know this?
1
Not at all
2
3
4
5
Perfectly
83
Q

Resting charge of a nerve

A

A charge is maintained across the axon membrane with the interior of the axon having a charge of −90 mV.

How well did you know this?
1
Not at all
2
3
4
5
Perfectly
84
Q

At what charge does a nerve depolarize?

A

When the resting membrane potential reaches −50 mV, the membrane depolarizes.

How well did you know this?
1
Not at all
2
3
4
5
Perfectly
85
Q

What causes an axon to be less negative?

A

Since there is a relatively greater concentration of K+ ions on the inside of the axon, there is a slow leak of K+ ions, which causes the inside of the axon to become less negative.

How well did you know this?
1
Not at all
2
3
4
5
Perfectly
86
Q

Actions on electrolytes of the ATP-dependent pump on a nerve

A

The ATP-dependent pump imports K+ and exports Na+, maintaining the normal resting membrane potential at −90 mV.

How well did you know this?
1
Not at all
2
3
4
5
Perfectly
87
Q

With chronic nerve compression, what changes occur to the axon?

A

With chronic nerve compression, the myelin sheath disintegrates and saltatory conduction between nodes of Ranvier ceases.

88
Q

A 25-year-oldman who is blind is brought to the emergency department two hours after sustaining a crush injury to the index finger of the dominant right hand at the flexion crease of the metacarpophalangeal joint. Physical examination shows full flexion and extension of the index finger. Sensation to pinprick is absent in the fingertip. Radiographs show no abnormalities. During surgical exploration, disruptions to the radial and ulnar digital nerves are noted. After debridement of the traumatized nerve ends, a 10-mm gap is noted in each digital nerve. Which of the following is the most appropriate management of each digital nerve?
(A)Bone shortening with primary repair
(B)Flexion of the index finger with primary repair
(C)Mobilization of nerve ends with primary repair
(D)Polyglycolic acid conduit between nerve ends
(E)Radial sensory nerve graft between nerve ends

A

(D)Polyglycolic acid conduit between nerve ends

In the first prospective, randomized multicenter evaluation of a bioabsorbable conduit for nerve repair, Weber et al. reported superior sensory recovery when polyglycolic acid conduits were used to repair nerve gaps greater than 8 mm compared to nerve graft repairs.

Although bone shortening has been described as a means of relieving tension on primary nerve repairs, this is clearly not the best choice when less morbid options for nerve reconstruction exist for the patient described. Another conduit available for small nerve gaps is vein, although that offers little mechanical resistance to kinking and collapse in larger defects.

Mobilization of the nerve with primary repair would likely place the nerve repair under some tension, which inhibits nerve regeneration. Management of short gaps by flexing joints to facilitate a tension-free repair can be problematic when the digit is mobilized because of scar formation.

Nerve grafts are an improvement over extensive nerve mobilization and limb positioning but are not without sequelae. Nerve graft harvest produces scarring, formation of neuroma, and loss of donor site function, in addition to increasing operating room and anesthesia time. The sural and posterior interosseous nerves are commonly used as grafts. Because of loss of sensitivity and potential for symptomatic neuromas, the radial sensory nerve is not commonly used as a graft.

89
Q

Nerve gap repairs in the digital nerves > 8 mm: Best results with what?

A

Superior sensory recovery when polyglycolic acid conduits were used, versus nerve graft repairs.

90
Q

Tension vs nerve repar

A

Tension inhibits nerve regeneration.

91
Q

Management of short gaps by flexing joints to facilitate a tension-free repair: Issue with this

A

Management of short gaps by flexing joints to facilitate a tension-free repair can be problematic when the digit is mobilized because of scar formation.

92
Q

Morbidity of nerve graft harvest

A

Nerve graft harvest produces scarring, formation of neuroma, and loss of donor site function, in addition to increasing operating room and anesthesia time.

93
Q

Radial sensory nerve as a source of nerve graft

A

Because of loss of sensitivity and potential for symptomatic neuromas, the radial sensory nerve is not commonly used as a graft.

94
Q
A 42-year-old woman comes to the office for evaluation of a severely painful mass on the upper right arm and numbness in the right forearm six months after undergoing decompression neuroplasty with submuscular transposition. This procedure was performed for progressively worsening compression of the dominant ulnar nerve at the elbow. She developed severe complex regional pain syndrome two weeks after the procedure. Physical examination shows a 0.5-cm mass proximal and central to the medial epicondyle that is adherent to the skin and extremely sensitive. Examination shows decreased sensation to light touch over the ulnar volar forearm. Following excision of the neuroma, which of the following is the most appropriate next step in management?
(A)Medial epicondylectomy
(B)Nerve stump implantation
(C)Stress occupational therapy program
(D)Topical lidocaine patch
(E)Ulnar nerve vein wrap
A

(B)Nerve stump implantation

The ulnar nerve neuroplasty is complicated by complex regional pain syndrome (CRPS) with an associated nerve injury. The medial antebrachial cutaneous nerve (MABC) of the forearm originates from the medial cord of the brachial plexus. The MABC derives from the C8 and T1 roots. In the mid arm, the MABC and the basilic vein pierce the deep fascia and provide sensation to the overlying ulnar volar forearm skin. The MABC can be injured during ulnar nerve at the elbow neuroplasty. If CRPS develops from an identifiable nerve injury, resolution is best achieved by identifyingand correcting the nerve injury. In the scenario described, excision and tensionless implantation into adjacent muscle or bone is the most appropriate option.

Medial epicondylectomy would not address the primary MABC neuroma, nor would wrapping the ulnar nerve with a vein graft. A stress occupational therapy program can be useful in CRPS without an identifiable nerve injury. With the injury described, however, the pain more likely would worsen. Topical transdermal anesthetic patches can alleviate but not resolve the nerve pain.

95
Q

The medial antebrachial cutaneous nerve (MABC) of the forearm originates from where in the brachial plexus?

A

The medial antebrachial cutaneous nerve (MABC) of the forearm originates from the medial cord of the brachial plexus.

96
Q

The medial antebrachial cutaneous nerve derives from which roots?

A

The MABC derives from the C8 and T1 roots.

97
Q

If complex regional pain syndrome develops from an identifiable nerve injury, resolution is best achieved how?

A

If CRPS develops from an identifiable nerve injury, resolution is best achieved by identifying and correcting the nerve injury.

98
Q

Nerve stump implantation

A

Excision and tensionless implantation into adjacent muscle or bone

99
Q
A 34-year-old man is brought to the emergency department one hour after sustaining a 7-cm laceration to the left wrist during a suicide attempt. The patient is unconscious but breathing. Physical examination shows a clean, linear radial laceration. Pulsatile arterial bleeding from the wound is noted. A tourniquet is applied to the upper left arm and inflated to 250 mmHg of pressure. Three hours later, the tourniquet is released, and the patient is brought to the operating room. The artery is ligated, and the tendons are repaired. Surgical exploration shows no nerve laceration. The next day, the patient has no sensation or motor function in the left hand. After three days, normal function returns. Which of the following Sunderland grades is the most appropriate classification for this injury?
(A)I
(B)II
(C)III
(D)IV
(E)V
A

(A)I

Nerve injuries can be classified with a system that was first described by Seddon and modified by Sunderland. The nerve injury described is classified as Sunderland grade I. Rapid nerve recovery is likely. The patient described had the tourniquet in place for longer than the generally accepted recommended period. Studies have shown that tourniquets should be released after two hours of use. Because of the prolonged use in the scenario described, the patient has sustained neurapraxia. When a tourniquet is used, the nerves and muscles are susceptible to injury. The nerves are particularly sensitive to pressure injury.

100
Q

Guideline for tourniquet inflation time

A

Two-hour use is a guideline; if additional time is needed, it is generally thought that five minutes of reperfusion is necessary for every 30 minutes of tourniquet time.

101
Q

In general Sunderland injuries:

A

A Sunderland grade I injury corresponds with neurapraxia. Complete recovery is expected. A grade II injury corresponds with axonotmesis and slow but complete recovery. Sunderland III, IV, and V injuries do not recover spontaneously. Axonotmesis and neurotmesis occur with Wallerian degeneration; therefore, significant regeneration often takes place only after surgery.

102
Q

Sunderland grading of nerve injuries?

A

Sunderland I, Neuropraxia || Complete recovery || Rapid rate of recovery
Sunderland II, Axonotmesis || Complete recovery || Slow rate of recovery
Sunderland III, Variable recovery || Slow rate of recovery
Sunderland IV || None || No recovery
Sunderland V, Neurotmesis || None || No recovery

103
Q

Sunderland I

A

Sunderland I, Neuropraxia || Complete recovery || Rapid rate of recovery

104
Q

Sunderland II

A

Sunderland II, Axonotmesis || Complete recovery || Slow rate of recovery

105
Q

Sunderland III

A

Sunderland III, Variable recovery || Slow rate of recovery

106
Q

Sunderland IV

A

Sunderland IV || None || No recovery

107
Q

Sunderland V

A

Sunderland V, Neurotmesis || None || No recovery

108
Q
A 37-year-old woman undergoing flexor tendon repair receives an axillary block. A tourniquet is used. Twenty minutes into the procedure, the patient says she has discomfort. Intolerance of the tourniquet is most likely caused by incomplete block of which of the following nerves?
(A)Axillary
(B)Median
(C)Musculocutaneous
(D)Radial
(E)Ulnar
A

(C)Musculocutaneous

The musculocutaneous nerve powers elbow flexion and forearm supination. Below the elbow, as the lateral antebrachial cutaneous nerve, it provides sensory function. An incompletely blocked musculocutaneous nerve is most likely to contribute to tourniquet intolerance. The axillary block is administered at the level of the terminal nerves of the brachial plexus. Both the musculocutaneous and axillary nerves leave the axillary sheath proximal to the level of injection. Because ofthis, they may be inadequately blocked with this anesthetic technique. Because the sensory distribution of the axillary nerve is above the level of the tourniquet application, it is less likely to contribute to tourniquet intolerance than an incompletely blocked musculocutaneous nerve. Multiple injection techniques, paresthesia elicitation, nerve stimulation, and ultrasound guidance increase the probability of a successful block. Administration of additional local anesthetic within the sheath, within the coracobrachialis muscle, or at the elbow can be used to block the musculocutaneous nerve if necessary. The interocostobrachial nerve conveys sensation from the medial aspect of the arm and must also be blocked for tourniquet tolerance.

109
Q

The musculocutaneous nerve innervates what?

A

The musculocutaneous nerve powers elbow flexion and forearm supination. Below the elbow, as the lateral antebrachial cutaneous nerve, it provides sensory function.

110
Q

Axillary nerve block: Which nerves are more likely to be inadequately anesthetized, and why?

A

The axillary block is administered at the level of the terminal nerves of the brachial plexus. Both the musculocutaneous and axillary nerves leave the axillary sheath proximal to the level of injection. Because of this, they may be inadequately blocked with this anesthetic technique.

111
Q

A 35-year-old man sustains a laceration of the volar aspect of the left index finger at the level of the mid phalanx while using a knife. There is smooth flexion and extension of the proximal interphalangeal and distal interphalangeal joints. Examination of the fingertip shows good color and capillary refill. Two-point discrimination is 5 mm on the ulnar aspect of the fingertip but is absent on stimulation of the radial aspect. On exploration, a laceration involving 45% of the cross-sectional area of the flexor digitorum profundus (FDP) tendon is noted. The radial digital nerve and artery are transected. The ulnar neurovascular bundle is intact. In addition to repair of the radial digital nerve, which of the following is the most appropriate management?
(A)Immediate protected mobilization
(B)Repair of the FDP tendon with core and epitendinous sutures and splint immobilization for two weeks followed by protected mobilization
(C)Repair of the FDP tendon with core sutures and splint immobilization for two weeks followed by protected mobilization
(D)Repair of the FDP tendon with epitendinous sutures and splint immobilization for four weeks followed by protected mobilization
(E)Splint immobilization for four weeks followed by protected mobilization

A

(A)Immediate protected mobilization

Tendon lacerations of less than 60% of the cross-sectional area of the tendon do not require repair. Attempts to repair the tendon may weaken it. Triggering may occur if the distal or proximal cut edge of the tendon catches on adjacent pulleys. If triggering occurs, trimming the cut edges of tendon, partial or complete pulley release, and repair of the flexor sheath can be performed. In cases of persistent triggering that persists despite these measures, the tendon can be repaired. In the absence of triggering, the tendon does not require treatment. Protected mobilization is performed to promote tendon gliding and strengthening. Despite early recommendations for several weeks of immobilization following digital nerve repair, the outcomes of digital nerve function following immediate protected mobilization are not statistically different than those managed with immobilization.

112
Q

Tendon lacerations of less than ___% of the cross-sectional area of the tendon do not require repair.

A

Tendon lacerations of less than 60% of the cross-sectional area of the tendon do not require repair.

113
Q

Adverse effects of tendon repair (why not do it when it’s not entirely necessary?)

A

-Attempts to repair the tendon may weaken it.

114
Q

When would you repair a tendon laceration of less than 60%?

A

Triggering may occur if the distal or proximal cut edge of the tendon catches on adjacent pulleys. If triggering occurs, trimming the cut edges of tendon, partial or complete pulley release, and repair of the flexor sheath can be performed. In cases of persistent triggering that persists despite these measures, the tendon can be repaired. In the absence of triggering, the tendon does not require treatment.

115
Q

Management of tendon laceration <60%

A

Protected mobilization is performed to promote tendon gliding and strengthening.

116
Q

Management of tendon laceration: Immobilization versus protected mobilization?

A

Despite early recommendations for several weeks of immobilization following digital nerve repair, the outcomes of digital nerve function following immediate protected mobilization are not statistically different than those managed with immobilization.

117
Q

A 17-year-old girl comes to the office because she has persistent wrist drop and weakness of digit extension three weeks after she fractured the intraarticular distal portion of the radius while snowboarding. Open reduction and internal fixation was performed under general anesthesia. A wide-cuff, pneumatic tourniquet was placed over cast padding; inflation pressure was 100 mmHg over the patient’s systolic blood pressure. Reduction was difficult, and tourniquet time was 180 minutes. Use of which of the following interventions is most likely to have decreased the risk of this complication?
(A)Elastic stocking rather than cast padding beneath the tourniquet
(B)Esmarch bandage as the tourniquet
(C)Narrow width tourniquet cuff
(D)Standard inflation pressure of 250 mmHg
(E)Tourniquet deflation/reperfusion period

A

(E)Tourniquet deflation/reperfusion period

The intervention most likely to decrease the incidence of nerve palsy in the patient described is providing a period of tourniquet deflation (―breather period‖) during which tissue acidosis may reequilibrate.

118
Q

A 20-year-old man comes to the office after he sustained transection of the ulnar nerve of the left arm during a rollover motor vehicle accident. Physical examination shows a 5-cm transverse laceration in the proximal ulnar aspect of his left forearm. A photograph of the laceration after suture removal is shown. Microscope-assisted epineural repair of the transection is performed. As function is restored over time, which of the following movements will most likely be last to return?
(A)Abduction of the small finger
(B)Adductionof the thumb
(C)Flexion of the metacarpophalangeal joint of the small finger
(D)Flexion of the proximal interphalangeal joint of the small finger
(E)Ulnar-sided flexion of the wrist

A

(B)Adductionof the thumb

The ulnar nerve is the terminal portion of the medial cord of the brachial plexus, after the medial head of the median nerve has separated from it, with fibers from C8–T1. It initially lies medial to the axillary artery and then to the brachial artery as it travels distally in the upper arm. It pierces the intermuscular septum and then follows the medial head of the triceps muscle to the groove between the olecranon process and the medial epicondyle. It gives off no branches in the arm. After the ulnar nerve passes through the cubital tunnel, it gives off articular branches and branches to the flexor carpi ulnaris (FCU) and the medial half of the flexor digitorum profundus (FDP). It travels between the two heads of the FCU and continues into the forearm between this muscle and the FDP.

In the distal half of the forearm, it is joined on its lateral side by the ulnar artery. Proximal to the wrist, the nerve gives off the dorsal sensory branch, providing sensation to the dorsal wrist and ulnar aspect of the hand. The ulnar nerve continues into the hand via Guyon’s canal, where it splits into a superficial sensory branch and a deep motor branch. The superficial branch supplies the palmaris brevis and the skin of the hypothenar eminence and digital nerves to the small and ulnar side of the ring finger. The motor branch passes between the abductor digiti minimi (ADM) and the flexor digiti minimi (FDM), with the deep branch of the ulnar artery, perforates the opponens digiti minimi and follows the deep volar arch across the interossei, and finally innervates the adductor pollicis (AD) and the deep head of the flexor pollicis brevis.

Based on its path, ulnar-sided flexion of the wrist (FCU) would be expected early, followed by abduction of the small finger (ADM) and flexion of the metacarpophalangeal joint (FDM). Interosseus muscle function would manifest as ability to abduct and adduct the fingers, followed last by adduction of the thumb (AD). Flexion of the proximal interphalangeal joints of the fingers is a function of the flexor digitorum superficialis, which is innervated by the median nerve.

119
Q

The ulnar nerve is the terminal portion of what?

A

The ulnar nerve is the terminal portion of the medial cord of the brachial plexus, after the medial head of the median nerve has separated from it,

120
Q

Fibers that contribute to the ulnar nerve?

A

Fibers from C8–T1.

121
Q

Ulnar nerve: Anatomical course in the upper extremity, proximal to the cubital tunnel

A

It initially lies medial to the axillary artery and then to the brachial artery as it travels distally in the upper arm. It pierces the intermuscular septum and then follows the medial head of the triceps muscle to the groove between the olecranon process and the medial epicondyle.

122
Q

The ulnar nerve gives off what branches in the arm?

A

It gives off no branches in the arm.

123
Q

Course of the ulnar nerve between the cubital tunnel and the distal forearm

A

After the ulnar nerve passes through the cubital tunnel, it gives off articular branches and branches to the flexor carpi ulnaris (FCU) and the medial half of the flexor digitorum profundus (FDP). It travels between the two heads of the FCU and continues into the forearm between this muscle and the FDP.

124
Q

Course of the ulnar nerve in the distal forearm until Guyon’s canal

A

In the distal half of the forearm, it is joined on its lateral side by the ulnar artery. Proximal to the wrist, the nerve gives off the dorsal sensory branch, providing sensation to the dorsal wrist and ulnar aspect of the hand. The ulnar nerve continues into the hand via Guyon’s canal.

125
Q

Course of the ulnar nerve distal to Guyon’s canal: Branches

A

The ulnar nerve continues into the hand via Guyon’s canal, where it splits into a superficial sensory branch and a deep motor branch.

126
Q

Ulnar nerve: Superficial branch

A

The superficial branch supplies the palmaris brevis and the skin of the hypothenar eminence and digital nerves to the small and ulnar side of the ring finger.

127
Q

Ulnar nerve: Motor branch

A

The motor branch passes between the abductor digiti minimi (ADM) and the flexor digiti minimi (FDM), with the deep branch of the ulnar artery, perforates the opponens digiti minimi and follows the deep volar arch across the interossei, and finally innervates the adductor pollicis (AD) and the deep head of the flexor pollicis brevis.

128
Q

Recovery of muscles innervated by the ulnar nerve

A

Based on its path, ulnar-sided flexion of the wrist (FCU) would be expected early, followed by abduction of the small finger (ADM) and flexion of the metacarpophalangeal joint (FDM). Interosseus muscle function would manifest as ability to abduct and adduct the fingers, followed last by adduction of the thumb (AD).

129
Q

A 37-year-old man comes to the emergency department because he has pain and swelling of the right hand as well as numbness of the fingertips two hours after undergoing CT scan of the abdomen because of abdominal pain. Physical examination shows lack of full active extension of the fingers and exquisite tenderness on passive adduction and abduction of the fingers. Pulses are intact. High-pressure infiltration of intravenous radiographic contrast medium into the hand is suspected. Which of the following is the most appropriate management?
(A)Administration of nonsteroidal anti-inflammatory drugs and follow-up evaluation in the office
(B)Elevation of the arm, application of ice, and re-evaluation in six hours
(C)Emergency fasciotomy
(D)Initiation of heparin therapy for anticoagulation
(E)Subcutaneous injection of calcium gluconate into the arm

A

(C)Emergency fasciotomy

The condition described is clearly a developing compartment syndrome that should be emergently decompressed in the operating room. The diagnosis should not be delayed until pulses are absent because absence of pulses is a late finding. Re-evaluation in six hours is not appropriate for the patient described because, by that time, irreversible muscle necrosis may occur, resulting in potential long-term disability. Similarly, having the patient follow up in the office is not appropriate. Initiation of heparin therapy for anticoagulation may be indicated in an ischemic event but is not appropriate in this scenario given the history and findings of the patient described. Subcutaneous injection of calcium gluconate is the treatment of choice for hydrofluoric acid burns, not for intravenous contrast infiltration.

In the case of a “non alert” patient, compartment pressures may be measured using an intracompartmental pressure monitor. Such devices are commercially available or may be fabricated. Interstitial compartment pressures of > 30 mmHg are consistent with compartment syndrome.

130
Q

Subcutaneous injection of calcium gluconate is the treatment of choice for:

A

Subcutaneous injection of calcium gluconate is the treatment of choice for hydrofluoric acid burns

131
Q

A 21-year-old man comes to the office because he has had absence of sensation of the right thumb since he underwent repair of the digital nerve on the opposition side of the digit one year ago. On physical examination, Tinel sign is absent at the surgical scar. After surgical exploration and debridement of the thumb, a 3-cm nerve gap is noted. Which of the following is the most appropriate method of digital reconstruction?
(A)Mobilization of the free nerve ends and primary repair
(B)Polyglycolic acid tube conduit
(C)Posterior interosseous nerve grafts
(D)Radial central nerve grafts
(E)Vein conduit

A

(C)Posterior interosseous nerve grafts

Repair or reconstruction of a digital nerve requires a tensionless technique. With tension, repair of the nerve will cause scarring and form a neuroma. If the gap is too large, a graft or conduit is required for a tensionless reconstruction. The terminal posterior interosseous nerve graft is ideal because it provides a nerve with similar caliber for the repair. Harvesting of this nerve avoids a donor deficit becauseat the level of the distal forearm wrist it only provides for wrist sensation.

Large nerve defects should not be reconstructed with vein or polyglycolic acid tube conduits. The radial sensory nerve is too large in diameter. Additionally, the sensory loss would be more problematic and would risk a painful neuroma.

132
Q

When is a graft or conduit necessary for a tensionless nerve reconstruction?

A

If the gap is too large, a graft or conduit is required for a tensionless reconstruction.

133
Q

Appropriateness of the terminal posterior interosseous nerve graft, specific to the digital nerve

A

The terminal posterior interosseous nerve graft is ideal because it provides a nerve with similar caliber for the repair.

134
Q

Donor site morbidity of terminal posterior interosseous nerve graft

A

Harvesting of this nerve avoids a donor deficit because at the level of the distal forearm wrist it only provides for wrist sensation.

135
Q

A 21-year-old woman who rows regularly with her university’s crew rowing club comes to the office because she has had pain on movement of the right thumb as well as pain and swelling of the distal forearm for the past month. She has not sustained trauma to the hand or arm. Physical examination shows tenderness and crepitus 4 cm proximal to the wrist and over the distal and radial aspects of the forearm. Slight discomfort is noted on ulnar deviation of the wrist with the thumb clasped in the palm. No grinding or crepitance of the thumb carpometacarpal joint is noted. Which of the following is the most appropriate initial step in management of this patient’s condition?
(A)Injection of a corticosteroid into the first dorsal compartment
(B)Injection of a corticosteroid into the third dorsal compartment
(C)Splinting of the carpometacarpal joint of the thumb
(D)Splinting of the wrist in extension
(E)Observation

A

(D)Splinting of the wrist in extension

The patient described has findings consistent with intersection syndrome, a tenosynovitis of the second dorsal compartment (common radial wrist extensors) associated with pain and swelling of the muscle bellies of the abductor pollicis longus (APL) and extensor pollicis brevis (EPB), 4 cm proximal to the wrist joint (ie, where the muscle bellies of APL and EPB cross extensor carpi radialis longus and extensor carpi radialis brevis). As in other stenotic conditions of the hand and wrist, it is associated with repetitive motions of the wrist and is frequently seen in athletes, especially rowers and weightlifters. Initial nonoperative treatment consists of rest, nonsteroidal anti-inflammatory drugs (NSAIDs), splinting of the wrist in 15 degrees of extension, and, in some patients, injection of a corticosteroid into the second dorsal compartment. For those with persistent pain, surgical release of the second dorsal compartment beginning at the wrist and extending proximal to the area of swelling may be necessary.

136
Q

Intersection syndrome:

A

Intersection syndrome is a tenosynovitis of the second dorsal compartment (common radial wrist extensors) associated with pain and swelling of the muscle bellies of the abductor pollicis longus (APL) and extensor pollicis brevis (EPB), 4 cm proximal to the wrist joint (ie, where the muscle bellies of APL and EPB cross extensor carpi radialis longus and extensor carpi radialis brevis).

137
Q

Intersection syndrome: Who gets it?

A

As in other stenotic conditions of the hand and wrist, it is associated with repetitive motions of the wrist and is frequently seen in athletes, especially rowers and weightlifters.

138
Q

Intersection syndrome: Management

A

Initial nonoperative treatment consists of rest, nonsteroidal anti-inflammatory drugs (NSAIDs), splinting of the wrist in 15 degrees of extension, and, in some patients, injection of a corticosteroid into the second dorsal compartment. For those with persistent pain, surgical release of the second dorsal compartment beginning at the wrist and extending proximal to the area of swelling may be necessary.

139
Q

De Quervain disease

A

De Quervain disease is a tenosynovitis of the first dorsal compartment,

140
Q

De Quervain disease: Presentation

A

Presents with wrist pain on the radial side aggravated by movement of the thumb. Local tenderness and swelling 1 to 2 cm proximal to the radial styloid and knife-like pain on Finklestein test (clasped thumb in palm while wrist is ulnarly deviated) is diagnostic.

141
Q

De Quervain disease most often occurs in what patient population?

A

most often in women aged 40–60 years.

142
Q

De Quervain disease: Treatment

A

In mild cases, rest, NSAIDs, and splinting of the wrist in gentle extension and the thumb widely abducted may be successful. Injection of a corticosteroid into the first dorsal compartment may be successful in 50%–80% of patients following one or two injections. If nonsurgical modalities are unsuccessful, then surgical release of the first dorsal compartment is successful .

143
Q

Tenosynovitis of the extensor pollicis longus: Importance of diagnosis

A

Tenosynovitis of the extensor pollicis longus (third dorsal compartment) is rare but requires early diagnosis and urgent operative treatment to prevent tendon rupture

144
Q

Tenosynovitis of the extensor pollicis longus: Presentation

A

It presents with pain, swelling, tenderness, and often crepitus at Lister’s tubercle.

145
Q

Tenosynovitis of the extensor pollicis longus: Management

A

Surgical treatment consists of third dorsal compartment release and tendon transposition radial to Lister’s tubercle. Injection of a corticosteroid is rarely indicated.

146
Q

Degenerative arthritis of the thumb most frequently affects:

A

Degenerative arthritis of the thumb most frequently affects the carpometacarpal joint.

147
Q
A 58-year-old woman who is a pianist comes to the officebecause she has had increasing clumsiness of the right hand for the past four months. She says the awkward movement of her hand makes it difficult for her to play the piano. Physical examination shows marked atrophy of the first dorsal interosseous muscle. Two-point discrimination is 3 to 4 mm in all fingers. The most likely cause of these findings is nerve compression at which of the following sites?
(A)Arcade of Struthers
(B)Carpal tunnel
(C)Guyon canal
(D)Ligament of Struthers
(E)Osborne ligament
A

(C)Guyon canal

148
Q

Isolated intrinsic motor weakness is most often a result of:

A

Isolated intrinsic motor weakness is most often a result of compression of the motor branch of the ulnar nerve with the ulnar tunnel, also referred to as the Guyon canal.

149
Q

The terminal portion of the ulnar nerve innervates:

A

The terminal portion of the nerve innervates the first dorsal interosseous muscle.

150
Q

Where is the ulnar nerve most commonly compressed?

A

At the elbow - cubital tunnel syndrome

151
Q

Release of the ulnar nerve at the elbow involves which structures?

A

Cubital tunnel
Osbourne ligament
Arcade of Struthers

152
Q

Cubital tunnel syndrome classically presents with:

A

Cubital tunnel syndrome classically presents with sensory changes within the small and ring finger followed by weakness in the intrinsic muscles and, in severe cases, weakness of the profundus tendon to the small and ring finger.

153
Q

A 13-year-old girl with cerebral palsy is brought to the office by her parents because she has had inability to voluntarily extend the dominant right wrist as well as weak grasp of the right hand since birth. On physical examination, when the fingers are held straight, the wrist can be brought to neutral. During grasp, there is ulnar deviation of the hand, and during extension of the fingers, there is radial deviation of the hand. Active extension of the fingers is intact, but active extension of the wrist is diminished. Dynamic electromyography confirms that function of the flexor carpi ulnaris (FCU) and flexor carpi radialis (FCR) tendons is intact. Which of the following is the most appropriate intervention for improvement in function of the hand?
(A) Fusion of the wrist
(B) Proximal row carpectomy
(C) Transfer of the superficialis tendon to the profundus tendon
(D) Transfer of the FCR tendon to the extensor carpi radialis brevis tendon
(E) Transfer of the FCU tendon to the extensor carpi radialis brevis tendon

A

(E) Transfer of the FCU tendon to the extensor carpi radialis brevis tendon

In this patient, there is a lack of wrist extension, which severely limits grip strength and hand function. In patients with cerebral palsy, poor grasp is often secondary to weak or absent wrist extension. Returning active wrist extension will increase the mechanical advantage of the digital flexor tendons and increase grasp strength. Active wrist extension may be created with a tendon transfer.

During grasp, this patient’s hand deviates to the ulnar aspect, suggesting a functioning flexor carpi ulnaris (FCU) tendon. Therefore, this tendon may be transferred to augment wrist extension. Muscles that fire during grasp work well when transferred for wrist extension, whereas muscles that act while the patient attempts to release objects work better for finger extension. Because this patient’s wrist tends to deviate radially with finger release, the flexor carpi radialis (FCR) would be better transferred as a finger extensor. It is important to ensure that the FCR is working, if the FCU is going to be transferred.

154
Q
A 25-year-old man who operates a jackhammer has had gradual onset of numbness and tingling for the past year. He says the discomfort awakens him every night and he is often clumsy when handling objects. Physical examination shows decreased sensation in the thumb, index finger, long finger, and radial ring finger. Weakness of thumb opposition and weakness of flexion of the thumb, index finger, and long finger are also noted. Which of the following is the most likely diagnosis?
(A) Anterior interosseous nerve palsy
(B) Carpal tunnel syndrome
(C) Cubital tunnel syndrome
(D) Posterior interosseous nerve palsy
(E) Pronator syndrome
A

(E) Pronator syndrome

The two most common problems of compression of the median nerve in the proximal forearm are anterior interosseous syndrome and pronator syndrome. Compression within different areas of the pronator teres results in either a pure motor deficit (anterior interosseous syndrome) or a mixed sensory and motor loss pattern (pronator syndrome). Forearm pain and numbness in the radial palm along with the thumb, index finger, long finger, and radial ring finger are common symptoms. Median intrinsic and extrinsic motor weakness also can be demonstrated on examination in pronator syndrome. The anterior interosseus nerve innervates the radial half of the flexor digitorum profundus (FDP), the flexor pollicis longus (FPL), and the pronator quadratus with sensory fibers to the radiocarpal, intercarpal, carpometacarpal, and radioulnar joints. Anterior interosseous nerve syndrome is characterized by impaired function of the FPL and FDP of the index finger with occasional involvement of the long finger and pronator quadratus. The thumb and index finger assume a classic position during pinch, with the distal interphalangeal (DIP) joint of the index finger extended and the proximal interphalangeal (PIP) joint of theindex finger in compensatory flexion while the interphalangeal (IP) joint of the thumb is hyperextended.

155
Q

The two most common problems of compression of the median nerve in the proximal forearm are:

A

The two most common problems of compression of the median nerve in the proximal forearm are anterior interosseous syndrome and pronator syndrome.

156
Q

What type of deficit is anterior interosseous syndrome versus pronator syndrome?

A

Anterior interosseous syndrome: Pure motor Pronator syndrome: Mixed sensory and motor loss pattern

157
Q

Compression within different areas of the pronator teres results in what?

A

Anterior interosseous syndrome or pronator syndrome

158
Q

Presentation of anterior interosseous syndrome

A

Forearm pain and numbness in the radial palm along with the thumb, index finger, long finger, and radial ring finger are common symptoms.

159
Q

Presentation of pronator teres syndrome

A

Forearm pain and numbness in the radial palm along with the thumb, index finger, long finger, and radial ring finger are common symptoms. Median intrinsic and extrinsic motor weakness also can be demonstrated on examination in pronator syndrome.

160
Q

What muscles does the anterior interosseous nerve innervate?

A

The anterior interosseus nerve innervates the radial half of the flexor digitorum profundus (FDP), the flexor pollicis longus (FPL), and the pronator quadratus

161
Q

Sensory of the anterior interosseous nerve?

A

Radiocarpal, intercarpal, carpometacarpal, and radioulnar joints.

162
Q

Clinical presentation of cubital tunnel syndrome

A

Cubital tunnel syndrome is caused by compression of the ulnar nerve at the elbow that manifests as numbness along the small and ulnar half of the ring fingers, often accompanied by weak grip, especially in activities in which torque is applied to a tool.

163
Q

Clinical presentation of posterior interosseous nerve palsy

A

Posterior interosseous nerve palsy is caused by compression of the nerve after the radial nerve bifurcates just proximal to the elbow and involves weakness and pain without a sensory component.

164
Q

Clinical course of posterior interosseous nerve palsy

A

Onset is often insidious and may not be noticed by the patient until late weakness of finger and wrist extension or a radial drift of the hand with wrist extension develops.

165
Q
A 28-year-old man comes to the office for evaluation of the nondominant left arm 12 weeks after he sustained a traction injury while working with heavy farm machinery. Physical examination shows complete loss of motor function in the ulnar nerve distribution, loss of sensation at the medial arm and forearm, and mild weakness of the left pectoralis major muscle. Nerve conduction testing of the ulnar nerve shows loss of motor conduction, but sensory conduction is intact. Which of the following is the most likely level of brachial plexus injury?
(A) Anterior cord
(B) Posterior cord
(C) Postganglionic root
(D) Preganglionic root 
(E) Terminal branch
A

(D) Preganglionic root

This patient has an avulsion injury to the lower roots of the brachial plexus. From proximal to distal, the levels of the brachial plexus are determined by its branching pattern and progress from spinal nerve roots (C5-T1), to trunks, to divisions, to cords, and to terminal branches. Loss of ulnar nerve function indicates damage to theulnar nerve itself, the medial cord, or the contributing C7, C8, and T1 roots. Loss of medial arm and forearm sensation indicate loss of the medial brachial and medial antebrachial cutaneous nerves, and weakness of the pectoralis muscle indicates loss of the medial pectoral nerve, which innervates the sternocostal head. The lateral pectoral nerve, which is intact in this case, innervates both the clavicular and sternocostal heads and maintains function of the pectoralis major muscle. Loss of motor conductivity with maintenance of sensory conductivity on nerve conduction testing indicates a disruption at the preganglionic nerve root level, with preservation of continuity between the sensory cell bodies in the dorsal root ganglion, the peripheral axons and the peripheral sensory organs, thus preventing Wallerian degeneration of the sensory neurons. The motor neuron cell bodies, however, are located in the ventral horn of the spinal cord, and are separated from their peripheral axons, leading to degeneration and loss of conductivity.

Disruption at the postganglionic nerve root level would result in separation of both sensory and motor cell bodies from their respective peripheral axons and subsequent degeneration. This would lead to loss of both motor and sensory conductivity on nerve conduction testing. Disruptions at the trunk level, cord level, or branch level would also lead to the same results. In addition, loss at the terminal branch level (ulnar nerve in this case), distal to the cord level, would preservethe medial brachial nerve and medial antebrachial nerve, and thus, sensation at the medial arm and forearm.

166
Q
A 14-year-old boy sustains a laceration of the distal forearm. Physical examination and wound exploration suggest flexor carpi ulnaris tendon and ulnar nerve involvement. The patient undergoes immediate microsurgical nerve repair with a nerve graft. Which of the following factors is most likely to predict a satisfactory outcome in this patient?
A) Age
B) Gender
C) Immediate repair
D) Ulnar nerve involvement
E) Use of nerve graft
A

A) Age

Multiple studies have evaluated outcomes of median and ulnar nerve repair after transection injury. A meta-analysis by Ruijs et al. confirmed that younger age, specifically under 16 years old, was associated with the highest chance of satisfactory recovery of motor function in both median and ulnar nerve injuries. Patient gender was not found to be a statistically significant factor in outcome. In the same analysis, median motor nerve injuries were found to have a better chance of recovery than ulnar motor nerve injuries. Timing influenced outcome, with delay of repair adversely affecting prognosis, and although the ideal window for repair was not able to be defined by this review, there is some evidence that immediate repair may result in worse outcomes. The use of nerve grafts did not significantly predict motor recovery in these injuries.

167
Q

A 65-year-old woman undergoes open reduction and internal fixation for the injury displayed in the x-ray study shown. Six weeks postoperatively, she has diffuse swelling, stiffness, and pain of the right upper extremity. Her skin appears shiny, and she has changes in the patterns of both hair and perspiration in the right upper extremity when compared with the left upper extremity. After injury, treatment with which of the following would most likely have decreased her risk for this complication?

A) Amitriptyline
B) Ascorbic acid
C) Gabapentin
D) Prednisone
E) Pregabalin
A

B) Ascorbic acid

Complex regional pain syndrome (CRPS) is chronic pain that persists in the absence of ongoing cellular damage and is characterized by autonomic dysfunction, trophic changes, and impaired function. In the perioperative period, the physiologic consequences of CRPS in the upper extremity contribute to or create one or more of the following: clinically significant osteopenia, delayed bony healing or nonunion, joint stiffness, tendon adhesions, arthrofibrosis, pseudo-Dupuytren palmar fibrosis, swelling, and atrophy. The reported incidence of CRPS is 5.5 to 26.2 per 100,000 person-years, and the prevalence is reported as 20.7 per 100,000 person-years. Women are more frequently affected than men, with a ratio of 3:1 to 4:1; the upper extremity is involved more frequently than the lower extremity; and fracture is the most common causative event. Incidence of CRPS after distal radius fracture has been reported to be between 22 and 39%.

In a double-blind, prospective, multicenter trial by Zollinger et al., 416 patients with 427 wrist fractures were studied for the effects of prophylactic vitamin C (ascorbic acid) on the risk of subsequent development of CRPS. Administration of 500 mg vitamin C daily was found to significantly decrease the incidence of CRPS in patients with distal radius fracture. The authors recommend treatment for 50 days.

The other four medications listed have all been reported for the treatment of patients diagnosed with CRPS. None have been reported to decrease the incidence of CRPS when used prophylactically

168
Q

A 30-year-old man is evaluated after sustaining a laceration of the left ulnar nerve at the level of the mid humerus. Exploration and direct repair of the ulnar nerve are performed immediately. Which of the following interventions is most likely to result in optimal recovery of hand function?
A) Anterior interosseus nerve transfer to the ulnar nerve
B) Dynamic splinting
C) Electrical stimulation of the ulnar nerve
D) Flexor digitorum superficialis tendon transfer to the A2 pulley
E) Wrist extensor tendon transfer to the lumbricals

A

A) Anterior interosseus nerve transfer to the ulnar nerve

High ulnar nerve injuries are plagued by poor recovery of intrinsic function even when the nerves are repaired under optimal conditions. To maximize recovery and restore intrinsic function in the hand, one of the most effective interventions is transfer of the motor branch of the anterior interosseus nerve to the motor branch of the ulnar nerve at the wrist. Recently, this has been shown to also be useful as a “babysitter” transfer, potentially retaining motor endplates in the small muscles of the hand while a primarily repaired ulnar nerve regenerates.

Dynamic splinting has played a role in radial palsies and is frequently employed to overcome joint contractures that are not fixed (with a hard endpoint). This would not assist in hand recovery in this setting.

Electrical stimulation performed by a hand therapist helps in retraining innervated muscles but will not improve function in the denervated small muscles of the hand.

Tendon transfers have been the mainstay of functional restoration when nerve repair is not possible and/or the timeframe that has elapsed is great enough to make successful reinnervation of the target muscles unlikely. Both the wrist extensor-to-lumbrical transfer and the flexor digitorum superficialis transfer have been used to correct the “clawing” associated with ulnar palsies.

169
Q
A 45-year-old right-hand–dominant man who is a tennis player is evaluated because of a 6-month history of pain in the right lateral elbow. He has pain when lifting objects, and the pain radiates to the forearm. Physical examination shows tenderness just distal and anterior to the lateral epicondyle. Which of the following muscles is most likely affected?
A) Brachioradialis
B) Extensor carpi radialis brevis
C) Extensor carpi radialis longus
D) Extensor carpi ulnaris
E) Extensor digiti minimi
A

B) Extensor carpi radialis brevis

The extensor carpi radialis brevis (ECRB) origin is the primary muscle involved in lateral epicondylitis. The undersurface is avascular, making it a potential site for degeneration and partial tears. The ECRB shares a common origin with the extensor carpi ulnaris, extensor carpi radialis longus, and brachioradialis. The extensor digiti minimi also originates from the lateral epicondyle and has been involved in some cases of lateral epicondylitis, but not as commonly as the ECRB

170
Q

Stellate ganglion blocks help alleviate the symptoms of complex regional pain syndrome in which of the following ways?
A) Helping repair nerve injuries
B) Increasing parasympathetic tone
C) Reducing sympathetic tone
D) Relaxing the muscles of the upper extremity
E) Releasing the body’s natural endorphins

A

C) Reducing sympathetic tone

Complex regional pain syndrome (CRPS) is a long-term pain condition that is believed to result from dysfunction in the central or peripheral nervous systems. CRPS is characterized by pain, swelling, or stiffness in the affected hand or extremity. The pain may be out of proportion to the injury that triggers it.

There are two types of CRPS. In type I, there is no identifiable nerve injury. In type II, there is an identifiable nerve injury. The name of this disorder was changed from reflex sympathetic dystrophy to CRPS because not all patients have increased sympathetic tone.

Stellate ganglion blocks may be used to anesthetize the stellate ganglion, which is a cluster of sympathetic nerves at the base of the neck, in an effort to decrease the overactivity of the sympathetic nerves seen in CRPS. The sympathetic nervous system has been implicated in the pathophysiology of CRPS, and consequently, sympathetic nervous system blockade is widely used to treat CRPS.

Sympathetic nervous system dysfunction is presumed to be an essential component of the syndrome, and sympathetic blockade has been recommended as early as possible to interrupt and reverse the process. The treatment effectively cuts the vicious cycle of pain, immobilization, and decreased joint motion.

Stellate ganglion blocks do not act to increase parasympathetic tone, relax muscles, release endorphins, or repair injured nerves.

171
Q

A 35-year-old woman who underwent surgical release of the first dorsal compartment 8 months ago is evaluated because of severe pain over the anatomical snuffbox of the right hand since the surgery. Physical examination shows focal tenderness over the region of the first dorsal compartment, a Tinel sign at the surgical scar, and numbness distal to the surgical scar. Medication includes gabapentin since the pain began, and the patient has attempted desensitization in hand therapy. Which of the following is the most appropriate next step in management?
A) Exploration silicone capping of the palmar cutaneous nerve
B) Initiation of oral pregabalin therapy
C) Iontophoresis with dexamethasone
D) Neurolysis and intramuscular placement of the superficial radial nerve
E) Phenol injection into the scar and compression glove therapy

A

D) Neurolysis and intramuscular placement of the superficial radial nerve

One potential complication from surgical treatment of de Quervain disease (first dorsal compartment release) is injury to branches of the radial sensory nerve. When such injury is suspected, treatment should consist of exploration and, if possible, repair of the injured nerve branch. When this is not possible, multiple surgical options are available; transposition of the injured radial sensory nerve into the brachioradialis has been one of the commonly employed treatment strategies. Alternatives include transposition of the nerve into bone or a vein, nerve stripping, and coverage of the injured nerve with vascularized tissue.

Both gabapentin and pregabalin are useful medical interventions for nerve pain. General guidelines suggest proceeding with surgical treatment for suspected neuromas if no improvement occurs after 6 months of oral therapy.

Iontophoresis with corticosteroids has no proven efficacy for neuroma therapy. Phenol has been employed for stump neuromas and for Morton’s neuromas; however, there is no evidence to support its use in non-amputation neuroma treatment.

Silicone capping, once advocated for treatment of end-neuromas, has fallen out of favor because of poor pain relief results and displacement/migration of the caps.

172
Q
A 43-year-old woman is evaluated for intrinsic wasting and paresthesias of the little and ring fingers. Which of the following is the most likely site of nerve entrapment?
A) Anconeus epitrochlearis
B) Arcade of Frohse
C) Lacertus fibrosis
D) Ligament of Struthers
E) Pronator teres
A

A) Anconeus epitrochlearis

Nerve entrapment is caused by the anconeous epitrochlearis. The scenario describes both motor and sensory signs and symptoms attributable to ulnar nerve compression. Ulnar nerve entrapment at the elbow is the second most common nerve entrapment neuropathy in the upper limb other than carpal tunnel syndrome in young adults. The most common cause is the ligament of Osborne. The anconeus epitrochlearis muscle, which is a congenital accessory muscle, arises from the medial epicondyle of the humerus and inserts at the olecranon process of the ulna. It can be found in normal elbows with an incidence of between 4 and 34%. It has been associated with ulnar nerve compression at the elbow.

The arcade of Frohse has been associated with posterior interosseous nerve entrapment. The lacertus fibrosis, pronator teres, and ligament of Struthers have been associated with median nerve compression. Note that the arcade of Struthers may be associated with ulnar nerve compression, but this option is not provided.

173
Q
A 36-year-old man comes to the office because of a 16-month history of diffuse pain of the posterior right shoulder. There is no history of trauma. Results of x-ray study are negative for an osseous injury. Physical examination shows atrophy isolated to the posterior scapular muscles. Motor and sensory examination of the right upper extremity shows weakness in shoulder external rotation with the arm adducted; no other abnormalities are noted. Which of the following nerves is most likely injured?
A) Axillary
B) Long thoracic
C) Spinal accessory
D) Suprascapular
E) Thoracodorsal
A

D) Suprascapular

The patient has an isolated palsy of the suprascapular nerve, the first branch off of the upper truck (C5, C6) of the brachial plexus. Causes can include trauma, ganglion cyst (supraspinous fossa), or direct compression of the nerve as it passes under the transverse scapular ligament. Symptoms include diffuse posterior shoulder pain, atrophy of the supraspinatus and infraspinatus muscles, and weakness in shoulder external rotation.

The axillary nerve comes off of the posterior cord and innervates the deltoid muscle and teres minor muscle; palsy would primarily impair shoulder abduction. The long thoracic nerve is composed of contributions from the C5-C7 roots and innervates the serratus anterior muscle; a deficit would lead to scapular winging. The thoracodorsal nerve (C6-C8), a branch of the posterior cord, innervates the latissimus dorsi muscle; a deficit would impair shoulder extension, adduction, and internal rotation. The spinal accessory nerve innervates the trapezius and sternocleidomastoid muscles. Injury to this nerve does not affect the periscapular musculature.

174
Q

A 44-year-old woman is evaluated because of a 6-month history of pain in her right upper chest and back, intermittent coolness in her right hand, and numbness and tingling of her right ring and little fingers. Results of the Adson test show a decreased radial pulse on the affected side, and the Roos test reproduces the patient’s symptoms on the affected side. Which of the following is the most appropriate next step?
A) Decompression of ulnar nerve at elbow
B) Noninvasive vascular study and electrodiagnostics
C) Resection of anterior and middle scalene muscles
D) Transaxillary resection of first rib
E) Observation

A

B) Noninvasive vascular study and electrodiagnostics

Three kinds of surgical procedures are employed to treat thoracic outlet compression syndrome (TOCS): transaxillary resection of the first rib, transcervical anterior and medial scalenectomies, and combined transaxillary first rib resection with immediate anterior and medial scalenectomies. This is the most complete procedure for total decompression of the thoracic outlet region. Because 70% of cases have soft-tissue involvement as the etiology of TOCS, current treatment includes transcervical anterior and middle scalenectomy in most TOCS cases.

Prior to any surgery, patients are treated conservatively with an exercise program for TOCS involving scalene stretching, first rib intercostal relaxation, nerve gliding, muscle relaxants, and pain patches for painful myofascial trigger points. Unfortunately, these conservative treatment modalities may yield only limited temporary help.

TOCS is usually classified in two groups. A neurogenic group comprises nearly 90% of all cases. This group usually has upper extremity pain, numbness, and tingling. A true vascular group comprises 10% of cases. Approximately 50% of patients still complain of coldness in the extremity. Approximately 40 to 50% of TOCS cases have concomitant peripheral nerve compression symptoms. Simple distal decompression of nerves will not usually lead to near-complete resolution of symptoms in cases of true TOCS.

There are two tissue groups that cause TOCS: soft tissue and osseous structures. The soft-tissue group includes anterior and middle scalene and their sheath, ligaments, and bands. This group comprises at least 70% of all TOCS cases because of congenital and acquired changes in the soft tissues. The osseous group comprises 30% or less of all TOCS cases and includes cervical rib, changes in the first rib, and clavicle due to injury.

Because TOCS can present with several different findings (including vascular and neurological compromise), it is advisable to work up these findings prior to committing to a treatment course. Noninvasive vascular studies and electrodiagnostics is the most reasonable first step in working up and treating these patients.

175
Q

A 51-year-old woman is evaluated because of numbness and tingling of the dorsal and palmar aspects of the left hand, extending to the ring and little fingers, with worsening symptoms at night. Physical examination shows weakness of finger abduction in the hand. Which of the following is the most likely electrodiagnostic finding?
A) Decreased median conduction velocity from above elbow to wrist
B) Decreased ulnar conduction velocity from above elbow to wrist
C) Decreased ulnar conduction velocity from below elbow to wrist
D) Prolonged median sensory latency from wrist to digit
E) Prolonged ulnar sensory latency from wrist to digit

A

B) Decreased ulnar conduction velocity from above elbow to wrist

This patient exhibits symptoms of ulnar nerve compression at the cubital tunnel. Cubital tunnel syndrome is characterized by numbness and tingling in the ulnar nerve distribution (ulnar side of hand, involving little finger and ulnar half of ring finger) and can lead to intrinsic weakness. Compression occurs at the level of the elbow, with slowing of nerve conduction across the area of compression. Electrodiagnostic findings in ulnar nerve compression consist of decreased ulnar conduction velocity in the segment from above elbow to the wrist. Comparison of conduction velocities between above elbow to wrist and below elbow to wrist may show a difference, with nerve conduction being faster when measured from below the elbow, as the area of compression is not traversed. Conduction velocities from below elbow to wrist should not be affected. Ulnar nerve compression at the cubital tunnel can be distinguished from compression at the Guyon canal, as symptoms in the dorsal hand are not involved during nerve compression at the Guyon canal (at the level of the wrist), because the dorsal sensory branch of the ulnar nerve branches proximal to the wrist.

Prolonged median sensory latency from wrist to digit is seen in carpal tunnel syndrome.

176
Q

A 32-year-old woman comes to the office for evaluation because of numbness of the left little finger 3 months after undergoing repair of a laceration of the left wrist sustained during an unsuccessful suicide attempt. Physical examination shows a healed laceration with a dysesthetic scar at the proximal wrist crease. A strong Tinel sign is present at the repair site. There is complete sensory loss of the little finger and no evidence of clawing. Motor function is intact. Wartenberg sign is absent. Two-point discrimination is greater than 15 mm. Which of the following is the most appropriate next step?
A) Microdissect the neuroma and identify motor fascicles with electrostimulation
B) Microdissect the neuroma and sural nerve graft fascicles
C) Resect the neuroma and direct repair with transposition
D) Resect the neuroma and repair with sural nerve grafts

A

A) Microdissect the neuroma and identify motor fascicles with electrostimulation

The patient described has an ulnar neuroma-in-continuity with intact motor function and no sensory regeneration. Mackinnon has described an electrostimulation technique where the proximal motor fibers are identified using nerve stimulation.

Resection of the neuroma, with or without transposition, is not appropriate because it would cause damage to intact nerve fascicles. Microdissection without nerve stimulation would also cause damage to intact fascicles.

177
Q

A 17-year-old boy is brought to the emergency department 5 hours after he sustained a stab wound to the left dorsal forearm. On physical examination, he is unable to extend the thumb and metacarpophalangeal joints. Exploration of the wound for repair of a presumed nerve injury is planned. Proper exposure of the nerve is between which of the following muscle groups?
A) Brachialis and triceps
B) Brachioradialis and extensor carpi radialis longus (ECRL)
C) Extensor carpi radialis brevis (ECRB) and ECRL
D) Extensor digitorum communis and ECRB
E) Flexor carpi radialis and pronator teres

A

D) Extensor digitorum communis and ECRB

Inability to extend the thumb and metacarpophalangeal joints generally suggests an injury to the radial nerve. Wrist extension can be preserved because of the preservation of the extensor carpi radialis longus muscle innervation. In the forearm, the radial nerve can be best approached directly between the extensor digitorum communis and the extensor carpi radialis brevis muscles. The approach between the brachialis and triceps muscles can identify the radial nerve in the upper arm. The interval between the flexor carpi radialis and pronator teres approaches the median nerve.

178
Q

Where is the radial nerve found in the forearm?

A

In the forearm, the radial nerve can be best approached directly between the extensor digitorum communis and the extensor carpi radialis brevis muscles.

179
Q

A 20-year-old man comes for evaluation 9 months after sustaining a stab wound to the left proximal upper arm. He did not seek medical attention at the time of the injury. Physical examination shows that he is unable to flex the left elbow with the forearm supinated. He is insensate to the lateral aspect of the upper arm and forearm. In addition to the ulnar nerve transfer to the biceps, which of the following nerve transfers is most appropriate to address this patient’s motor deficit?
A) Median nerve to brachialis
B) Musculocutaneous nerve to brachioradialis
C) Posterior interosseous nerve to triceps
D) Radial nerve to pronator teres
E) Ulnar nerve to flexor carpi radialis

A

A) Median nerve to brachialis

The patient described has sustained a laceration of the musculocutaneous nerve. With this injury, the patient would be able to flex the elbow with the forearm in a pronated position using the brachioradialis, which is innervated by the radial nerve.

Due to the proximal level and amount of time that has passed since the injury, repair with grafting of the musculocutaneous nerve may not be advisable as the axons may not reach the motor end plates of the biceps and brachialis muscles before degeneration.

Use of fascicles from the median nerve, ulnar nerve, and both nerves has been described to restore elbow flexion. Fascicles are transferred distally in the upper arm directly to the nerve branch to the brachialis and/or biceps muscle. Distal coaptation allows donor axons to reach the target muscles more rapidly, and is more appropriate for this patient whose status is nearly 1 year post injury. A fascicle from the ulnar nerve was transferred to the nerve to the biceps (blue dot) and a fascicle from the median nerve was transferred to the nerve to the brachialis (green dot).

The musculocutaneous nerve is injured and cannot be used as a donor. The posterior interosseous nerve does not branch off the radial nerve until distal to the elbow; it would not be able to reach proximal enough to coapt to the nerve to the brachialis or biceps. The pronator teres does not flex the elbow; the median nerve is not injured. The flexor carpi radialis does not flex the elbow; the median nerve is not injured.

180
Q

An 18-year-old man comes to the office because he has “drooping” of the right shoulder and inability to abduct it beyond 30 degrees. He underwent exploration and vascular repair 4 months ago after sustaining a stab wound to the right side of the neck. Photographs are shown. Which of the following surgical transfer techniques is most likely to restore deltoid function in this patient?
A) C5 to C6 nerve root
B) Intercostal to musculocutaneous nerve
C) Partial ulnar nerve to musculocutaneous nerve
D) Radial nerve fascicle to axillary nerve
E) Suprascapular to axillary nerve

A

D) Radial nerve fascicle to axillary nerve

Of the nerve transfers listed, only the transfer of a triceps branch to the axillary nerve will restore innervation to the deltoid muscle and provide shoulder abduction. Although debate remains over whether nerve graft repair (if possible) or nerve transfer provides better outcomes for shoulder abduction, both techniques are frequently employed in the setting of upper trunk brachial plexus injury.

The location of the injury (based on scar and mechanism) makes the injury likely distal to the nerve roots; in addition, root to root transfers are not typically part of the brachial plexus reconstruction ladder.

The suprascapular nerve is often the recipient nerve for partial transfers from cranial nerve XI (spinal accessory nerve) in order to provide shoulder stability. In this pattern of injury and based on the photos demonstrating atrophy of the supraspinatus and infraspinatus, the suprascapular nerve could not act as a donor nerve.

Intercostal nerve and partial ulnar nerve transfers to the musculocutaneous nerve are both frequently used to restore elbow flexion in the setting of upper trunk injuries.

181
Q

In relation to the pronator teres muscle, which of the following is the most likely location of the median nerve in the proximal third of the forearm?
A) Between the superficial (humeral) and deep (ulnar) heads of the pronator teres muscle
B) Deep to the superficial (humeral) and deep (ulnar) heads of the pronator teres muscle
C) Superficial to the superficial (humeral) and deep (ulnar) heads of the pronator teres muscle
D) Through the deep (ulnar) head of the pronator teres muscle
E) Through the superficial (humeral) head of the pronator teres muscle

A

A) Between the superficial (humeral) and deep (ulnar) heads of the pronator teres muscle

The course of the median nerve is relatively consistent. Just proximal to the elbow, at the medial epicondyle, there is a constant relationship of the median nerve, brachial artery, and the biceps tendon. From medial to lateral, the mnemonic, MAT, describes the relationship (Median nerve, brachial Artery, and biceps Tendon). In the cubital fossa, the nerve dives deep to the lacertus fibrosus, lying anterior to the brachialis muscle and medial to the brachial artery. The nerve enters the forearm between the superficial (humeral) and deep (ulnar) heads of the pronator teres muscle. As the nerve passes through the muscle bellies, it crosses the ulnar artery anteriorly, from medial to lateral, separated from the artery by the deep head of the pronator teres. After emerging from the pronator teres, the median nerve passes deep to an arch created by the two heads of the flexor digitorum superficialis. The nerve continues distally in the forearm between the flexor digitorum superficialis and flexor digitorum profundus. The nerve usually becomes superficial approximately 5 cm proximal to the wrist, emerging between the flexor digitorum superficialis and flexor carpi radialis, dorsal and slightly radial to the palmaris longus tendon.

182
Q

A 24-year-old woman comes to the office because of severe thenar atrophy 3 years after failed repair of a low median nerve laceration. Tendon transfer to restore thumb opposition is planned. Optimal transfer would restore which of the following thumb functions?
A) Palmar abduction, extension, supination
B) Palmar abduction, flexion, pronation
C) Palmar abduction, flexion, supination
D) Radial abduction, extension, pronation
E) Radial abduction, flexion, supination

A

B) Palmar abduction, flexion, pronation

Injury to the median nerve, either by laceration or compression, results in thenar atrophy and loss of thumb opposition. Tendon transfer is the only reliable technique to restore thumb function in the face of severe, long-standing atrophy. Thumb opposition is a composite movement comprised of palmar abduction, flexion, and pronation. Opposition positions the thumb for grasp, but is not synonymous with it. There are numerous tendon transfer procedures described to improve thumb opposition (e.g., palmaris longus, abductor digiti minimi, flexor digitorum superficialis), but the most effective improve each of the three components. Thumb extension, supination, and radial abduction (in the plane of the hand) are not movements involved in opposition.

183
Q

Movements that comprise thumb opposition

A

Thumb opposition is a composite movement comprised of palmar abduction, flexion, and pronation.

184
Q
Which of the following is the earliest one might expect to find electromyographic changes after suspected median nerve damage during carpal tunnel release?
A) 1 Week
B) 3 Weeks
C) 5 Weeks
D) 7 Weeks
A

B) 3 Weeks

Patients with nerve injuries can be evaluated by nerve conduction velocities and electromyography (EMG). Abnormal conduction velocities are associated with decreased amplitude, decreased velocity, and increased latency. Sensory nerve latency above 3.5 ms and/or motor nerve latency above 4.5 ms are considered abnormal. Muscle changes assessed by EMG are typically altered later (2 to 3 weeks after injury) in the clinical course and consist of the presence of fibrillation potentials and decreased motor unit potential recruitment. Though the sensitivity of these studies may be somewhat low, at around 66% when using conduction velocity and latency, the specificity has been reproducibly near 95%.

185
Q
An otherwise healthy 58-year-old man comes to the office because of numbness of all fingers of the left hand. He says he first noticed symptoms after a cross-country drive 6 weeks ago. On physical examination, the thumb, index, and long fingers show sensitivity to the 2.83 Semmes-Weinstein monofilament. The ring and little fingers show sensitivity to the 3.22 monofilament. The little finger is held in an abducted position. Abductor pollicis brevis muscle strength is normal. First dorsal interosseous muscle strength is diminished. Which of the following anatomical structures is the most likely cause of these findings?
A) Arcade of Frohse
B) Lacertus fibrosus
C) Ligament of Struthers
D) Osborne ligament
E) Transverse carpal ligament
A

D) Osborne ligament

This scenario depicts a patient with ulnar nerve compression. Often patients who present with compression neuropathies give a history of numbness of all fingers; however, careful physical examination will show sensory abnormalities only in the anatomical location of the compression. The patient has weakness of the ulnar nerve innervated intrinsic muscles, the first dorsal interosseous muscle, but retains strength in the abductor pollicis brevis muscle. The diagnosis of ulnar nerve compression is suggested. The most common sight of ulnar nerve compression is at the elbow. The anatomical causes of all the nerve compression at the elbow are the arcade of Struthers, the medial intramuscular septum, the bony cubital tunnel, Osborne ligament, an anconeus epitrochlearis muscle, and the origin of the flexor carpi ulnaris muscle. The ligament of Struthers, lacertus fibrosus and the transverse carpal ligament are anatomical sites of compression of the median nerve. The arcade of Frohse is a site of compression of the radial nerve.

186
Q
A 48-year-old woman comes to the office because of burning pain and stiffness in the right hand 6 weeks after treatment of a distal radius fracture. She says she has had difficulty sleeping and that she has discomfort despite taking narcotics. Physical examination shows a shiny appearance of the right hand, decreased range of motion of the fingers, and hypersensitivity to light touch. X-ray studies show good alignment of the fracture. Which of the following tests is the most appropriate to evaluate this patient's condition?
A) Bone scan
B) CT scan
C) Digital subtraction angiography
D) Lymphoscintigraphy
E) Ultrasonography
A

A) Bone scan

This patient exhibits symptoms of complex regional pain syndrome (CRPS). The persistence of physiological changes after surgery or injury can lead to debilitating consequences. This condition is characterized by persistent pain, cold intolerance, autonomic dysfunction, and trophic changes. Patients may show swelling, stiffness, difficulty sleeping, and persistent pain out of proportion to the normal postoperative course that may be relieved incompletely by narcotics.

CRPS is a clinical diagnosis without a single definitive test, and is divided into two types: type I, which occurs without identifiable nerve involvement (also known as reflex sympathetic dystrophy); and type II, which has identifiable nerve involvement (causalgia). It is more common in people who smoke and in women. Pain in CRPS can be either sympathetically mediated or sympathetically independent

The diagnosis of CRPS involves history, physical examination, and diagnostic testing. Although no specific test is pathognomonic, triple-phase bone scans are helpful in adding credence to the diagnosis. First- and second-phase bone scans may show asymmetric flow and autonomic dysfunction, while the third phase demonstrates increased periarticular uptake in multiple joints of the affected extremity.

A variety of treatment modalities have been employed in addressing CRPS. These range from therapy modalities such as range of motion, stress loading, and desensitization to pharmacologic interventions with anticonvulsants or antidepressants. Stellate ganglion blocks or autonomic nerve blocks may be helpful in sympathetically mediated pain, and nerve stimulation (either transcutaneous or at the spinal cord level) can also be employed. Often, multiple modalities are used concurrently and in sequence. Peripheral nerve decompression may be helpful in resolving symptoms related to CRPS type II.

CT scans can be used to assess bony alignment in fractures and are helpful in the evaluation of articular anatomy.

Digital subtraction angiography is useful for evaluation of vasculature and circulation

Lymphoscintigraphy is used to analyze lymphatic drainage in cases of lymphedema.

Ultrasonography can be performed to assess venous outflow and look for deep venous thrombosis.

187
Q

A 35-year-old man comes to the emergency department with a humerus fracture. On examination, he is unable to extend his wrist, fingers, and thumb. Which of the following nerves is most likely injured?

A) Axillary
B) Median
C) Musculocutaneous
D) Radial
E) Ulnar
A

D) Radial

This patient has a radial nerve injury, which can occur with humerus fractures. The radial nerve innervates the wrist extensors, extensor carpi radialis longus (ECRL) and extensor carpi radialis brevis (ECRB); the thumb extensors, extensor pollicis longus (EPL) and extensor pollicis brevis (EPB); and the finger extensors, extensor digitorum communis (EDC). These radial nerve injuries are usually managed with a period of observation and therapy until a potential neurapraxia resolves. Early evidence of muscle reinnervation would be evident with improved function of the ECRL, followed by the ECRB, then the finger and thumb extensors. If the patient does not regain any function by 3 to 6 months, exploration and nerve repair or tendon transfers to restore lost function can be considered. The standard tendon transfers considered for radial nerve function loss include the pronator teres to the ECRB (wrist extension), flexor carpi ulnaris to the EDC (finger extension), and the palmaris longus to the EPL (for thumb extension). A median nerve deficit would result in loss of flexion of wrist, fingers, thumb, and loss of palmar sensation and not typical after humerus fractures. An ulnar nerve injury could present with loss of hand intrinsic function and loss of sensation of the small finger. The musculocutaneous nerve innervates the biceps and would result in loss of elbow flexion. The axillary nerve is not injured with humerus fractures.

188
Q

A 40-year-old, right-hand–dominant man comes to the office because of minimal sensation in the tips of the digits of the right hand and severe pain in the volar aspect of the right wrist. He sustained lacerations of the median and ulnar nerves, radial and ulnar arteries, and all volar flexor tendons 11 months ago when he punched a window. Each of the structures was repaired primarily within hours of injury. On examination, the hand is well perfused. Percussion at the location of the dot in the photograph shown produces severe, painful paresthesia, which radiate distally. Sharp touch sensation is minimally present at the tips of the thumb, index, and long fingers and the radial side of the ring finger; light touch sensation is absent. Which of the following is the most appropriate next step in treatment?
A) Administration of tacrolimus
B) Excision repair with a sural graft
C) Neurolysis and conduit wrap
D) Reassessment in 3 months
E) Transfer of the extensor indicis proprius

A

B) Excision repair with a sural graft

The patient has a neuroma of the median nerve after laceration and repair. At 11 months out from injury, he would be expected to have improving light touch at the fingertips. The combination of this and the severe pain with percussion of the volar wrist indicates that very few axons have crossed the repair site. Observing the patient for additional time is unlikely to yield improved recovery. At surgical exploration, the patient had a large neuroma at the repair site. A photograph is shown.

Tacrolimus is an immune modulating agent commonly used in solid organ transplants. It has been investigated in animals and clinical trials in humans and shown some effectiveness in improving nerve regeneration across the repair. It is not currently used in humans outside of clinical trials. Also, in all trials of tacrolimus use, the medication was started at the time of repair, not 11 months later.

Nerve conduits have been used to repair short-gap nerve injuries. Although there are reports of successful nerve regeneration of gaps greater than the original indication of 1.5 cm, a gap of 6 cm is well beyond the limits of what a conduit can bridge.

Transfer of the extensor indicis proprius tendon would restore thumb opposition and use a tendon not involved in the original injury. However, this would not address the patient’s principal complaint of pain and lack of sensation.

189
Q

A 30-year-old man is evaluated because he is unable to abduct or externally rotate his shoulder or flex his elbow 4 months after he was involved in a high-speed motor vehicle collision. Physical examination shows numbness of the lateral upper arm and forearm. Which of the following nerve transfers is most appropriate to restore external rotation of the shoulder?
A) C7 ipsilateral root to anterior division of upper trunk
B) Medial pectoral nerve to medial cord
C) Phrenic nerve to long thoracic nerve
D) Radial nerve to axillary nerve
E) Spinal accessory nerve to suprascapular nerve

A

E) Spinal accessory nerve to suprascapular nerve

Examination findings indicate a C5-6 avulsion or very proximal upper trunk injury. External rotation of the shoulder is provided by the supraspinatus and infraspinatus muscles. Multiple techniques for harvest of the distal spinal accessory nerve to transfer to the suprascapular nerve have been described. This transfer would restore external rotation.

Transfer of the radial nerve to the axillary nerve will provide deltoid and possibly teres major innervation. This will improve shoulder function overall but will not restore external rotation. The phrenic nerve is commonly used to provide donor motor axons, but the long thoracic nerve goes to the serratus anterior.

Contralateral C7 transfer can be used to innervate the upper trunk. Ipsilateral C7 would not be used because the C5 and C6 roots were destroyed in the injury. Also, coaptation to the anterior division of the upper trunk would be distal to the origin of the suprascapular nerve.

The medial pectoral nerve can also be used for donor motor axons. It is limited by its relatively short reach. Coaptation to the medial cord of the brachial plexus would innervate the ulnar nerve, which is not injured in this patient.

190
Q
A 53-year-old man is evaluated because of a 5-month history of numbness and tingling of the right ring and little fingers. He says that symptoms are worse at night and when his elbow is flexed while holding a phone to his ear. Physical examination shows weakness of abduction of the fingers of the right hand. Which of the following structures is the most likely cause of this patient's symptoms?
A ) Arcade of Frohse
B ) Flexor digitorum superficialis
C ) Ligament of Struthers
D ) Osborne band
E ) Transverse carpal ligament
A

D ) Osborne band

The structure most likely to be responsible for the condition described is Osborne band.

The patient described exhibits symptoms of ulnar nerve compression at the cubital tunnel. Cubital tunnel syndrome is characterized by numbness and tingling of the ulnar nerve distribution (ulnar side of hand, involving little finger and ulnar half of ring finger) and can lead to intrinsic weakness. The ulnar nerve passes posterior to the medial epicondyle and travels between the medial epicondyle and the olecranon at the level of the elbow. In the region of the cubital tunnel, a set of thick fascial fibers known as Osborne band or ligament forms the roof of the tunnel and can compress the ulnar nerve. A number of additional sites can contribute to ulnar nerve compression, including the arcade of Struthers, the medial intermuscular septum, anconeus epitrochlearis, and the fascia of the flexor carpi ulnaris or flexor pronator musculature.

The arcade of Frohse is a fibrous arch that comes from the lateral epicondyle, which can compress the radial nerve.

The flexor digitorum superficialis can cause compression of the median nerve in the forearm.

The ligament of Struthers is an anatomic variant which arises from the supracondylar process of the humerus. It can cause proximal median nerve compression.

The transverse carpal ligament is responsible for median nerve compression at the carpal tunnel.

191
Q
A 50-year-old woman has significant tenderness of the residual tip of the index finger 8 weeks after undergoing amputation of the fingertip. Physical examination shows significant stump tenderness. Which of the following therapy modalities is most appropriate for desensitization of the amputation stump?
A ) Functional electric stimulation
B ) Immobilization
C ) Kinesiology tape
D ) Semmes-Weinstein monofilaments
E ) Vibration
A

E ) Vibration

Vibration is a modality that may be used in therapy to desensitize an amputation stump neuroma. The treatment consists of vibratory stimulation applied to the periphery of the sensitive area and then gradually moving toward the center. Some additional methods used in therapy to treat neuroma pain include desensitization, massage, and transcutaneous nerve stimulation. Functional electric stimulation is similarly not a therapeutic modality. Kinesiology taping is used for edema control and comfort. It would not be used on an amputated stump. Semmes-Weinstein monofilaments are used to quantify sensation to fine touch and are not used for desensitization.

192
Q

Arcade of Frohse and nerve compression

A

The arcade of Frohse is a fibrous arch that comes from the lateral epicondyle, which can compress the radial nerve.

193
Q

Flexor digitorum superficialis and nerve compression

A

The flexor digitorum superficialis can cause compression of the median nerve in the forearm.

194
Q

Ligament of Struthers and nerve compression

A

The ligament of Struthers is an anatomic variant which arises from the supracondylar process of the humerus. It can cause proximal median nerve compression.

195
Q

Transverse carpal ligament and nerve compression

A

The transverse carpal ligament is responsible for median nerve compression at the carpal tunnel.

196
Q

Osbourne band and nerve compression

A

In the region of the cubital tunnel, a set of thick fascial fibers known as Osborne band or ligament forms the roof of the tunnel and can compress the ulnar nerve.

197
Q
A 23-year-old man is brought to the emergency department after being stabbed with a knife in a fight. The patient appears alert and in no distress. Physical examination shows a 2-cm puncture wound just above the midclavicular line. Decreased strength with wrist flexion, weakness of index, long, ring, and little finger flexion at the metacarpophalangeal (MCP) and proximal interphalangeal (PIP) joints, and difficulty with palmar abduction against resistance are noted. Finger, thumb, and wrist extension is intact. Elbow flexion and extension are normal. Sharp injury to which of the following structures is most likely in this patient?
A ) Lateral cord
B ) Medial cord
C ) Median nerve
D ) Middle trunk
E ) Ulnar nerve
A

B ) Medial cord

The patient described demonstrates median and ulnar nerve weakness after a sharp injury to the brachial plexus. The radial nerve function appears to be preserved.

The medial cord gives fibers to the medial pectoral nerve, the medial brachial and antebrachial cutaneous nerves, the median nerve, and the ulnar nerve. Therefore, this is the appropriate response.

The lateral cord is not appropriate because although that structure does give fibers to the median nerve, it does not branch to the ulnar nerve. The lateral cord terminates in the lateral pectoral nerve, musculocutaneous nerve, and the median nerve. Also of note, as the patient has intact elbow flexion, the musculocutaneous nerve is presumably intact in the scenario described.

The median nerve in isolation is not appropriate as well for similar reasons. This patient has median and ulnar nerve deficits.

The middle trunk contains fibers that go to the radial and median nerves, but not the ulnar. This is not appropriate, therefore, as well.

The posterior cord gives off the radial nerve, axillary nerve, subscapular nerves, and the thoracodorsal. It does not contribute to the median or ulnar nerve.

At the midclavicular line, the ulnar nerve has not yet formed.

198
Q

The medial cord gives fibers to:

A

The medial cord gives fibers to the medial pectoral nerve, the medial brachial and antebrachial cutaneous nerves, the median nerve, and the ulnar nerve.

199
Q

The lateral cord gives fibers to:

A

The lateral cord terminates in the lateral pectoral nerve, musculocutaneous nerve, and the median nerve.

200
Q

The middle trunk gives fibers to:

A

The middle trunk contains fibers that go to the radial and median nerves, but not the ulnar.

201
Q

The posterior cord gives fibers to:

A

The posterior cord gives off the radial nerve, axillary nerve, subscapular nerves, and the thoracodorsal.

202
Q

A 25-year-old woman comes for evaluation because she has difficulty gripping things with her dominant right hand. She sustained a stab wound to the left forearm 6 months ago. Physical examination shows a Tinel sign present over the ulnar aspect of the mid volar forearm with a well-healed laceration. A neuroma in-continuity is identified intraoperatively. The neuroma is resected, and a 3-cm nerve gap remains. Which of the following is the most appropriate management?

A ) Bridge the gap with a synthetic nerve conduit
B ) Dissect the nerve both proximally and distally with an end-to-end neurorrhaphy
C ) Proceed to tendon transfers to assist with grip
D ) Reconstruct the nerve defect with a lateral antebrachial cutaneous nerve graft
E ) Sural nerve cable grafting

A

E ) Sural nerve cable grafting

The sural nerve is a common source of nerve graft material. It is formed from the medial cutaneous sural nerve that originates from the tibial nerve.

Synthetic nerve conduit repair of nerves has not proven effective in gaps larger than 2 cm and is less effective in motor nerve defects.

A tension-free repair is the goal for the nerve anastomoses. When an acute laceration occurs, primary end-to-end neurorrhaphy is indicated. When the treatment is delayed and a neuroma exists, the type of repair is dictated by the size of the nerve gap. Nerve grafting is indicated to bridge a defect when more than 10% elongation of the nerve would be necessary to bridge the gap.

Tendon transfers would not be indicated at this point.

The lateral antebrachial cutaneous nerve is the distal continuation of the musculocutaneous nerve. Given the size of the defect and the need for at least two cable grafts, this nerve would not be appropriate.

203
Q

An 8-year-old girl is brought to the office for evaluation because her right forearm is severely contracted at the wrist and fingers. Upon questioning, the patient’s parents note that she was treated 1 year ago for a humerus fracture. Physical examination shows a pronated forearm, flexed wrist, hyperextended metacarpophalangeal (MCP) joints, and flexed proximal (PIP) and distal interphalangeal (DIP) joints. She can make a full active fist. Which of the following is the most appropriate management?

A ) Exploration at the level of the humerus fracture with neurolysis and/or repair of the radial nerve
B ) Exploration of the forearm, neurolysis of the median and ulnar nerves, and muscle slide technique performed
C ) Exploration of the upper arm with median and ulnar nerve neurolysis
D ) Flexor carpi radialis to extensor carpi radialis brevis tendon transfer

A

B ) Exploration of the forearm, neurolysis of the median and ulnar nerves, and muscle slide technique performed

Volkmann described the sequelae of compartment syndrome following a supracondylar fracture of the humerus in a child with the development of a severely contracted and functionless forearm. The forearm is typically fixed in pronation, the wrist is flexed, and the hand is postured in the ?claw? position with the MCP joints hyperextended and the PIP and DIP joints flexed. The hand is usually insensate. Treatment depends on the severity of the deformity. Moderate contractures are treated with exploration and release of both the median and ulnar nerves and a tendon lengthening procedure (muscle slide). This condition is not due to a radial nerve injury; therefore, exploration of the fracture site and neurolysis of the radial nerve are not indicated. Tendon transfer is not appropriate because it does not address the nerve compression. Revascularization with a bypass graft is not needed because it is no longer ischemic.

204
Q
A 36-year-old man is brought to the emergency department 2 hours after being stabbed in the hand with a knife. Physical examination shows a wound over the dorsum of the first web space. Exploration with administration of a local anesthetic agent is planned. Which of the following is the most appropriate landmark for injection of the local anesthetic?
A ) A1 pulley
B ) Flexor carpi radialis
C ) Medial epicondyle
D ) Pisiform
E ) Radial styloid
A

E ) Radial styloid

The most appropriate location for injection of local anesthesia is the radial styloid.

Wrist blocks are useful for providing anesthesia during hand surgery. Sensation is blocked, while preserving movement of the extrinsic musculature. The sensory supply to the hand is provided by the median, ulnar, and radial nerves. The median nerve innervates the volar radial aspect of the hand, including thumb, index, long, and the radial half of the ring finger. The median nerve can be blocked as it travels between the flexor carpi radialis and palmaris longus tendons. Sensation to the thenar eminence is provided by the palmar cutaneous branch, which divides about 5 to 7 cm proximal to the wrist crease. The ulnar nerve innervates the ulnar aspect of the hand, including the ulnar aspect of the ring finger and the little finger. At the level of the wrist, the ulnar nerve lies radial to the flexor carpi ulnaris tendon, and ulnar to the ulnar artery. The dorsal cutaneous branch of the ulnar nerve branches proximal to the wrist crease and travels dorsally to innervate the ulnar aspect of the dorsum of the hand. It can be blocked as it crosses the region of the ulnar styloid. The radial nerve provides sensation to the dorsal radial aspect of the hand, including the first web space. The superficial radial nerve divides into several branches in the region of the radial styloid and can be blocked by subcutaneous infiltration over the radial styloid proximal to the anatomical snuffbox. This patient has a wound in the dorsal first web space, which is innervated by the radial nerve; hence, injection at the radial styloid level will provide anesthesia to the area.

A digital block can be performed with local injection in the region of the A1 pulley but would result in anesthesia too distal to the area of interest in this case.

The median nerve can be anesthetized by injecting local anesthetic just ulnar to the flexor carpi radialis tendon.

Injection just posterior to the medial epicondyle would result in blockade of the ulnar nerve at the elbow.

The ulnar nerve can be blocked at the level of the wrist by injecting radial to the flexor carpi ulnaris tendon and pisiform bone.

205
Q

Location for median nerve block

A

The median nerve can be blocked as it travels between the flexor carpi radialis and palmaris longus tendons. Sensation to the thenar eminence is provided by the palmar cutaneous branch, which divides about 5 to 7 cm proximal to the wrist crease.

206
Q

Location for ulnar nerve block

A

At the level of the wrist, the ulnar nerve lies radial to the flexor carpi ulnaris tendon, and ulnar to the ulnar artery. The dorsal cutaneous branch of the ulnar nerve branches proximal to the wrist crease and travels dorsally to innervate the ulnar aspect of the dorsum of the hand. It can be blocked as it crosses the region of the ulnar styloid.

207
Q

Location for radial nerve block

A

The radial nerve provides sensation to the dorsal radial aspect of the hand, including the first web space. The superficial radial nerve divides into several branches in the region of the radial styloid and can be blocked by subcutaneous infiltration over the radial styloid proximal to the anatomical snuffbox.

208
Q

Consultation is requested for a 7-year-old girl because of intravenous infiltration of a chemotherapeutic agent in the dorsal forearm. Physical examination shows firmness and swelling of the forearm and pain on passive flexion of the wrist. Which of the following is the most appropriate initial management?
A) Administration of an antidote
B) Doppler sonography of the forearm
C) Liposuction and saline flush of the affected area
D) Measurement of compartment pressures
E) Surgical excision and grafting

A

D) Measurement of compartment pressures

Extravasation usually remains localized, yet some patients develop necrotic problem wounds. Often initially underestimated, the extent of injury can declare itself widely with time. Compartment syndrome in an extremity extravasation should be initially ruled out either by clinical assessment or direct measurement of compartment pressures. Tissue loss can include skin, muscle, tendon, nerve, vasculature, and/or joint.

Given the variable amount of soft-tissue involvement, early conservative therapy is recommended. Immediate discontinuation of the infusion at the affected site is paramount and should not be overlooked. Aspiration, liposuction, wound excision, debridement, grafts, flaps, and antidote administration have all been described in the management of extravasation injury.

209
Q
A 24-year-old man comes to the office because of numbness and difficulty moving his ring and little fingers 5 months after cutting his upper arm on broken glass. Current physical examination shows inability to abduct and adduct the ring and little fingers. Sensation to light touch is diminished. Following exploration and resection of a painful, traumatic neuroma, there is a 5-cm gap in the ulnar nerve proximal to the elbow. Which of the following is the most appropriate management to restore intrinsic muscle function?
A) Cadaveric nerve allografting
B) Nerve transfer
C) Sural nerve grafting
D) Use of nerve conduit
E) Vascularized nerve grafting
A

B) Nerve transfer

The most appropriate management for restoration of intrinsic muscle function is nerve transfer.

In nerve injuries resulting in complete transection of the nerve, wallerian degeneration occurs at the site of transection, and Schwann cells in the distal nerve segment undergo apoptosis. With prolonged denervation, decreased regenerative ability with limitation in motor recovery is noted. Optimal functional recovery is dependent upon adequate reinnervation of the motor end plates and target muscles by regenerating motor axons. Over time, loss of target motor end plates via degeneration and fibrosis and replacement of muscle fibers by fat cells occur.

Nerve regeneration occurs at a rate of approximately 1 mm daily or 1 inch monthly. In a high injury to the ulnar nerve, the distance from the proximal motor axons to the intrinsic musculature precludes timely reinnervation, and intrinsic recovery is generally poor. Reinnervation of the muscle ideally should be completed within 12 to 18 months following injury to allow for recovery.

In the patient who has had the delayed symptoms and high ulnar nerve injury described, the time to recovery of intrinsic function will be greater than 2 years if the injury is reconstructed directly. This estimate is based on the elapsed time and distance to the target muscles.

Nerve transfer involves the use of a noncritical or expendable donor motor nerve to reinnervate a missing function. The selection of an available motor nerve donor that is closer to the target muscle can decrease the time needed for reinnervation of the muscle and help to ensure recovery before irreversible changes occur. In the scenario described, the distal portion of the anterior interosseous nerve can be used as a donor nerve to reinnervate the ulnar motor branch. Transfer of the distal anterior interosseous nerve to the motor branch of the ulnar nerve will provide motor neurons in a more distal location to reinnervate the intrinsic muscles in the desired time frame.

Nerve grafting is the most appropriate management to bridge a nerve gap when direct repair is not possible. This would be indicated if the circumstances dictated that muscle reinnervation could occur in an adequate or timely fashion, such as in a more recent injury or in a nerve gap that is closer to the target muscles. Nerve grafting may be performed for sensory recovery, but motor reinnervation is unlikely to occur in the scenario described. The sural nerve provides a good source of nerve autograft that is long (up to 40 cm) and of reasonable diameter (2 to 3 mm), with minimal donor site morbidity.

A variety of artificial nerve conduits have been developed to avoid the need to harvest nerve grafts. Nerve conduits of polyglycolic acid and collagen have been developed to bridge nerve gaps; however, recovery is not as effective as compared with autogenous nerve grafts. Typically, conduits are used for sensory nerves in noncritical areas. Gaps of up to 2 to 3 cm can be bridged.

There has been recent interest in processed preserved nerve allografts which are obtained from cadaveric sources. Studies suggest that allograft nerves may regenerate motor neurons better than nerve conduits, but autologous nerve grafting remains the gold standard. Typically, nerve allografts are more useful for short segmental nerve gaps.

A vascularized nerve graft allows transfer of the blood supply along with the nerve. This decreases the need for revascularization from the surrounding tissues and may be useful for grafting long nerve gaps with badly scarred or irradiated beds.

210
Q
A 13-year-old boy is brought to the office because he has difficulty opening his hand and extending his fingers. History includes release of the forearm compartments to treat a pulseless hand following a supracondylar humerus fracture 2 years ago. On physical examination, passive extension of the fingers is restricted when the wrist is fully extended; it improves with full wrist flexion. Which of the following muscles is the most likely cause of the limitation described?
A) Flexor carpi radialis
B) Flexor carpi ulnaris
C) Flexor digitorum profundus
D) Flexor digitorum superficialis
E) Lumbricals
A

C) Flexor digitorum profundus

The most likely cause of the restricted finger extension described is fibrosis of the flexor digitorum profundus muscle. The patient exhibits Volkmann ischemic contracture as a complication of late treatment (over 24 hours from the time of initial ischemia) of arterial compromise associated with the fracture. The muscle groups at the greatest risk during these ischemic episodes are within the deep flexor compartment of the forearm. This risk occurs because the arterial supply is relatively distant from the usual site of occlusion and because this compartment is relatively less distensible. In the scenario described, the flexor digitorum profundus and flexor pollicis longus are at the greatest risk.

Superficial muscle groups such as the flexor carpi radialis, flexor carpi ulnaris, and the flexor digitorum superficialis typically recover some function and do not lead to contractures in the forearm. Likewise, the small muscles of the hand, such as the lumbricals, tend to be less severely injured than the deep compartment of the forearm.

211
Q

A 24-year-old man comes to the office because he says the ring and little fingers of his right hand “catch” when he puts his hand in his pocket and that he “pokes” himself in the eye when washing his face. History includes repair of a complete transection of the right ulnar nerve at the wrist 1 year ago. On physical examination, he is unable to extend the interphalangeal joints of the ring and little fingers when the metacarpophalangeal joints are flexed. Photographs are shown. Which of the following tendon transfers is the most appropriate management?
A) Extensor indicis proprius (EIP) to adductor
B) EIP to extensor digiti minimi
C) EIP to first dorsal interosseous
D) Flexor digitorum superficialis (FDS) of the little finger to A2 pulley
E) FDS of the little finger to lateral band

A

E) FDS of the little finger to lateral band

Of the tendon transfer choices offered, only the FDS transfer to the lateral band (of both the ring and little fingers) will correct the loss of interphalangeal joint extension described, thereby diminishing the tendency for the flexed/abducted finger to catch on pocket edges.

The clinical scenario and photographs demonstrate failure of the intrinsic muscle function to return following a low ulnar nerve repair. The deformities demonstrated include ulnar clawing of the little finger primarily, abduction of the little finger (Wartenberg sign), hyperflexion of the interphalangeal joint of the thumb, and atrophy of the intrinsics (especially notable in the hypothenar eminence on the lateral view). Of these deformities, the patient is bothered primarily by the little finger deformity.

Correction of clawing can be achieved actively or passively. Patients who can extend the interphalangeal joints while hyperextension of the metacarpophalangeal joints is blocked (Bouvier test) can achieve correction of clawing with active or passive transfers. Active transfers attempt to re-create the normal function of the intrinsics by directing pull through the lateral bands. Passive transfers re-create the intrinsic function of metacarpophalangeal joint flexion (similar to externally blocking hyperextension) but do not extend the interphalangeal joints.

EIP transfers are useful for correction of the lateral pinch functions of the intrinsic minus hand. An EIP transfer to the adductor tendon re-creates the thumb component of lateral pinch, while the EIP transfer to the first dorsal interosseous tendon would improve the index function in pinch.

EIP transfer to the extensor digiti minimi is one method used to reduce hyperabduction of the little finger. This would not correct the flexion deformity at the level of the proximal interphalangeal joint.

FDS transfer to the A2 pulley provides a passive transfer, which, based on the patient’s inability to extend the interphalangeal joints during the Bouvier test, would not correct the deformity.

212
Q
A 25-year-old right-hand dominant man is brought to the emergency department after sustaining a stab wound to the right arm in a bar fight. Physical examination shows a 2 × 1-cm laceration over the antecubital fossa. He is unable to flex the interphalangeal joint of the thumb and the proximal interphalangeal joint of the index finger. Which of the following nerves is most likely injured?
A) Lateral antebrachial cutaneous
B) Median
C) Musculocutaneous
D) Radial
E) Ulnar
A

B) Median

Median nerve palsy is marked by the inability to oppose the thumb or flex the thumb at the interphalangeal joint. The inability to flex the index finger at the proximal interphalangeal joint is also noted. The lateral antebrachial cutaneous nerve provides sensory innervation to the lateral aspect of the arm. The median antebrachial cutaneous nerve innervates the skin of the anterior and middle surfaces of the forearm to the level of the wrist. This nerve does not innervate any muscles. Radial nerve palsy is marked by the inability to extend the fingers, thumb, and wrist. Patients with radial nerve palsies have difficulty grasping objects. The results of tendon transfers to restore function in patients with radial nerve palsies are among the best and most predictable outcomes. Ulnar nerve palsy symptoms include a ?claw? deformity, with flexion deformities of the ring and little fingers. In later stages, profound muscle wasting of the both hypothenar eminence and the first web space is seen.

213
Q

A 55-year-old man with bilateral carpal tunnel syndrome comes to the outpatient surgical unit for elective surgical intervention of the dominant right hand. He will be the tenth procedure of the day for the surgeon performing the operation. The surgeon favors an open technique; he has performed 150 carpal tunnel operations since finishing his hand fellowship 3 years ago. Which of the following is most likely to increase the risk of wrong-site surgery?
A) The elective nature of the procedure
B) Only one surgeon is involved in the operation
C) The procedure will be performed using an open technique
D) The surgeon has a high volume of cases scheduled for the same day
E) The surgeon has only been practicing independently for 3 years

A

D) The surgeon has a high volume of cases scheduled for the same day

A Joint Commission review of a series of sentinel events identified a number of factors contributing to the increased risk of wrong-site surgery, such as emergency cases; unusual physical characteristics, including morbid obesity or physical deformity; unusual time pressures to start or complete the procedure; unusual equipment or setup in the operating room; multiple surgeons involved in the case; and multiple procedures being performed during a single surgical visit. A large series of wrong-site hand surgeries showed an increased rate of wrong-site surgery with increasing surgeon age and experience, and a direct correlation with increasing surgical case volumes.

214
Q
A 34-year-old man is brought to the emergency department 2 hours after sustaining injuries to the right wrist when he punched a glass window. Surgical exploration shows a complete laceration of the median nerve at the level of the wrist. A 1-cm gap between the proximal and the distal stumps of the nerve is noted. Which of the following treatments is most likely to provide the best functional outcome?
A) Multistrand nerve grafting
B) Nerve transfer
C) Nerve transposition
D) Primary epineurial repair
E) Single-strand nerve grafting
A

D) Primary epineurial repair

The need for nerve grafting is dependent upon many parameters, such as the length of the gap, the excursion of the nerve, the wound bed, and vascularity, among others. For clean, sharp injuries with nerve gaps measuring less than 1 cm in a large peripheral nerve such as the median, most authors agree that primary repair of the nerve results in the best outcome. Autologous nerve grafting should be reserved for cases in which there is tension on the nerve ends with primary repair. Both multistrand nerve grafting and single-strand nerve grafting produce similar outcomes and are inferior to primary repair. Nerve transfer would only be considered if there were no proximal nerve to repair to the distal nerve. Nerve transposition would only be appropriate for gaining length in the ulnar nerve, where the switch from the extensor side of the elbow to the flexor side results in increased relative length in the nerve.

215
Q

Multistrand vs single strand nerve grafting

A

Both multistrand nerve grafting and single-strand nerve grafting produce similar outcomes and are inferior to primary repair.